AP Environmental Science Final

¡Supera tus tareas y exámenes ahora con Quizwiz!

Directions: The following questions(s) refer to the following graph. At which time after clear-cutting does the ecosystem exhibit the greatest species richness? A 50 years B 100 years C 150 years D 200 years E 250 years

50 years

Based on the second law of thermodynamics, how much biomass of phytoplankton would be needed to produce 11 kilogram (kgkg) of large fish in this food web? A 1g B 10g C 100kg D 1,000kg

1,000g

The LD50 for arsenic in humans is 13mg/kg. Which of the following approaches should be used to calculate the number of grams of arsenic it would take to reach the LD50 in a 140-pound person? [Note: 1kg=2.2pounds1 A 140lb×2.2lb1kg×1kg13mg×1g1,000mg B 140lb×1kg2.2lb×13mg1kg×1g1,000mg C 13mg1kg×50×140lb×1kg2.2lb D 13mg1kg×1kg2.2lb×140lb1

140lb×1kg2.2lb×13mg1kg×1g1,000mg

What is the population doubling time in years for a country with an annual growth rate of 3.5 percent? A 0.5 B 3.5 C 20 D 24.5 E 70

20

A sample of radioactive waste has a half-life of 10 years and an activity level of 2 curies. After how many years will the activity level of this sample be 0.25 curie? A 10 years B 20 years C 30 years D 40 years E 80 years

30 years

Researchers are evaluating the waste treatment facility located on the Moose River in the northeastern United States. They are sampling the river and its tributaries at several locations, as shown on the map below. Water from which of the following locations on the map would best serve as a control group for the study? A B C D

A

Which of the structures in the diagram is the site where the nuclear power plant accidents at Three Mile Island and Chernobyl occurred? A B C D

A

Which soil horizon is best developed in areas with dense vegetation, such as forests and grasslands, since the horizon consists of decomposing leaves and other plant material? A B C D

A

Which of the following best shows the process of evolution? A A lizard's color becomes brown as it sits on a log. B A bear goes into hibernation. C A plant loses its leaves in a drought. D A population of mosquitoes develops resistance to a pesticide. E A population of foxes increases as more prey becomes available.

A population of mosquitoes develops resistance to a pesticide.

Based on the data in the graph, which location showed an annual average grain yield in 1991-1993 that was approximately 2.3 times the yield in 1961-1963 ? A Africa B Latin America C Asia D Near East

Asia

Which of the following types of organisms are required to complete the nitrogen cycle, including the process of denitrification? A Bacteria B Producers C Consumers D Fungi

Bacteria

The country with the highest population density is A Australia B Bangladesh C China D Ethiopia E India

Bangladesh

Directions: Select the choice that best fits each statement. The following question(s) refer to the following energy sources. Biomass Wind Tidal energy Nuclear fission Sunlight The source whose use is a direct cause of deforestation A Biomass B Wind C Tidal energy D Nuclear fission E Sunlight

Biomass

In the majority of less developed countries, the major source of energy for domestic use is which of the following? A Oil B Coal C Biomass D Nuclear E Geothermal

Biomass

Which of the following terrestrial biomes has the highest primary productivity per unit area? A Desert B Tropical rain forest C Boreal forest D Savanna

Boreal Forest

Which of the following is a greenhouse gas that is produced by domestic livestock? A NO2 B CH4 C O3 D CO E SO2

CH4

Which of the following shows the approximate concentration of CO2 , N2 , and O2 in dry air? A CO2 78% N2 < 1% O2 21% B CO2 43%N2 < 1O2 56% C CO2 36% N2 8%O2 56% D CO2 10%N2 70%O2 20% E CO2 < 1%N2 78%O2 21%

CO2 < 1% N2 78% O2 21%

This biome is found in shallow waters off the coastline and is Earth's most biologically diverse marine biome. A Coral reefs B Freshwater wetlands C Open ocean D Rivers

Coral reefs

Which of the following best describes an advantage of constructing habitat corridors? A Corridors connect habitats separated by fragmentation and connect wildlife populations. B Corridors prevent invasive species from traveling between protected habitats. C Corridors provide a safe passage for vehicles and allow humans to travel within wildlife preserves. D Corridors increase the amount of protected area that is not affected by humans.

Corridors connect habitats separated by fragmentation and connect wildlife populations.

Which soil horizon is the least weathered horizon and contains rock that is starting to break down into soil? A B C D

D

Of the following organisms, which occupies the lowest trophic level? A Spider B Deer C Lion D Hawk E Snake

Deer

Directions: The following questions(s) refer to the locations indicated on the diagram below.This region has been the largest petroleum exporter for the past four decades. A B C D E

E

Which of the following best describes an environmental problem associated with hydrologic fracturing, or fracking? A Fracking has led to an increase in the production of oil and natural gas, lowering energy prices. B Fractures in the shale will close up once the fracking fluid is removed unless fine particles, like sand, are introduced to hold them open. C The products of fracking release more carbon dioxide and sulfur compounds into the atmosphere than other fossil fuels. D Fracking fluids can escape from the drill hole and contaminate surface water and groundwater.

Fracking fluids can escape from the drill hole and contaminate surface water and groundwater.

This biome contains a nutrient-rich environment created by falling leaves and trapped organic materials from the large trees, and it provides the ecosystem service of filtering pollutants from water. A Coral reefs B Freshwater wetlands C Open ocean D Rivers

Freshwater wetlands

Characteristics that tend to increase the risk of a species becoming endangered include which of the following? Having a very limited distribution Being a specialist at the end of a long food chain Having a small population size A I only B II only C III only D II and III only E I, II, and III

I, II, and III

Factors that affect the total fertility rate of a human population include which of the following? Cultural traditions Government policies and economic incentives Education level and economic opportunities for females A I only B II only C I and II only D II and III only E I, II, and III

I, II, and III

Which of the following major reservoirs of freshwater are missing from the diagram? A Fossil fuels B Rocks C Ice caps D Algae

Ice Caps

Directions: Select the choice that best fits each statement. The following question(s) refer to the following possible relationships between organisms in an ecosystem. (A) Commensalism (B) Parasitism (C) Mutualism (D) Predation (E) Competition Exemplified by bees consuming nectar and carrying pollen from one flower to another A Commensalism B Parasitism C Mutualism D Predation E Competition

Mutualism

Which of the following is the most abundant gas by volume in the atmosphere? A CO2 B N2 C CH4 D O2

N2

Which of the following biogeochemical cycles includes the processes of buffering ocean pHpH and photosynthesis, as shown in the diagram? A The carbon cycle B The nitrogen cycle C The phosphorus cycle D The potassium cycle

The carbon cycle

A student is setting up a model to study the carbon cycle and natural sources of carbon dioxide. They create a micro environment that contains soil, leaf debris, an earthworm, heterotrophic bacteria, and a sun lamp. What natural source of carbon dioxide and major metabolic process was left out of the model? A There is no representation of decomposition in the model environment. B There is no representation of cellular respiration in the model environment. C There is no representation of photosynthesis in the model environment. D There is no representation of ingestion in the model environment.

There is no representation of photosynthesis in the model environment

Which of the following terrestrial biomes has the highest net primary productivity? A Desert B Tropical rain forest C Taiga D Savanna

Tropical rain forest

Compared to a coal-fired power plant that produces the same amount of energy, a nuclear power plant generates more A CO2 B SO2 C fly ash D particulates E thermal pollution

thermal pollution

Small populations of animals, such as those typically found on islands, are prone to extinction because A they contain a high degree of genetic variability B they tend to have more predators C their habitats are more stable D they are more vulnerable to unpredictable events E they are usually generalist species

they are more vulnerable to unpredictable events

Overgrazing of public land by privately owned livestock is an example of the A tragedy of the commons B right of eminent domain C rule of seventy D principle of manifest destiny E swapping of debt for nature

tragedy of the commons

The process in the hydrologic cycle in which water vapor is released from leaves into the atmosphere is called A infiltration B transpiration C sublimation D reflection E percolation

transpiration

Natural gas is often priced in units of dollars per therm. One therm equals 100,000BTUs. A certain family uses 600 therms of energy to heat its home annually using a natural gas furnace that is 80 percent efficient. The family is considering replacing its current furnace with one that is 96 percent efficient. Assuming the cost of a therm is$0.30, which of the following would be the family's annual savings in the cost of home heating? A $23.04 B $28.80 C $138.24 D $172.80

$28.80

In many parts of the United States, homeowners use a significant portion of their energy budget to heat their home in the winter. It has been determined that lowering a thermostat by 3°F in the winter can reduce the energy costs by about 10 percent in many homes. An individual homeowner lowers their thermostat by 3 degrees exactly halfway through the billing period. For the month that the thermostat was lowered, the heating bill was $95.How much money did the family save in the month in which they lowered the thermostat setting halfway through the billing period? A $2.85 B $5.00 C $9.50 D $10.00

$5.00

If an incandescent lightbulb used for lighting has an efficiency rating of 5 percent, then for every 1.00 joule of electrical energy consumed by the bulb, which of the following is produced? A 1.05 joules of light energy B 1.05 joules of heat energy C 0.95 joules of light energy D 0.05 joules of light energy E 0.05 joules of heat energy

0.05 joules of light energy

Corn can be grown at a rate of 1,000 kcal per hectare, and it takes 20 times more land to produce beef than it does to produce corn. Which of the following equations would correctly calculate the amount of land needed to produce 10,000 kcal of beef? A 10,000 kcal corn×20 hakcal beef×100 ha B 1 ha1,000 kcal corn×20 kcal corn1 kcal beef×10,000 kcal beef C 20 kcal beef1 kcal corn×10,000 kcal10 ha×20 beef1 corn D 20 ha1,000 kcal beef×1,000 kcal corn1 ha E 10 ha10,000 kcal beef×10,000 cal corn1 ha×1 ha corn20 ha beef

1 ha1,000 kcal corn×20 kcal corn1 kcal beef×10,000 kcal beef

A home uses ten 100-watt lightbulbs for five hours per day. Approximately how many kilowatt-hours of electrical energy are consumed in one year by using the lightbulbs? A 365 B 1,825 C 5,000 D 10,500 E 365,000

1,825

If a country has a crude birth rate of 24 per 1,000 and a crude death rate of 8 per 1,000, the natural annual percent increase of its population is A 0.6% B 1.6% C 3% D 16% E 32%

1.6%

The half-life of radon gas is approximately four days. Four weeks after the introduction of radon into a sealed room, the fraction of the original amount remaining is closest to A 1/ 2 B 1/ 8 C 1/ 32 D 1/ 64 E 1/128

1/128

The greatest amount of fresh water is found in which of the following? A The atmosphere B Estuaries C Lakes D Rivers and streams E Polar ice caps and glaciers

Polar ice caps and glaciers

The graphs above show the population size and distribution of beak sizes in a finch population on the island of Daphne Major in the Galapagos in 1976 and 1978. Smaller-beaked finches are better suited for eating grasses and small seeds. Larger-beaked finches are better suited for cracking larger seeds and nuts. In 1977 a drought took place and the vegetation of the island was affected by the lack of rain. Which of the following was the number of finches in the population with a beak size of 10.2 mm in 1978? A 8 B 12 C 40 D 65

12

If the population of a country grows at a rate of approximately 5 percent per year, the number of years required for the population to double is closest to A 5 years B 10 years C 15 years D 25 years E 35 years

15 years

Based on the laws of thermodynamics, which of the following is the applied mathematical routine used to estimate the biomass of the mice in the pyramid? A 18,705kg×10 B 18,705kg/10×100 C 18,705kg×0.10×0.10 D 18,705kg/0.10

18,705 kg×0.10×0.10

Directions: The following question(s) refer to the graph below showing the percent contribution of major fuel sources that have supplied energy for the United States for the past 150 years. The category labeled "Other" includes nuclear power, hydroelectricity, solar and wind power, and other alternative sources of energy.In what year was the fraction of energy supplied by coal the greatest? A 1880 B 1910 C 1940 D 1970 E 2000

1910

Directions: The following question(s) refer to the graph below showing the percent contribution of major fuel sources that have supplied energy for the United States for the past 150 years. The category labeled "Other" includes nuclear power, hydroelectricity, solar and wind power, and other alternative sources of energy.In approximately what year did oil and gas become the dominant fuels for the United States? A 1880 B 1910 C 1940 D 1970 E 2000

1940

What is the population doubling time in years for a country with an annual growth rate of 3.5 percent? A 0.5 B 3.5 C 20 D 24.5 E 70

20

In 1997 the World Resources Institute estimated the world's proven oil reserves to be 1,000 billion barrels and the ultimately recoverable reserves to be 2,000 billion barrels. The table below shows the world consumption of oil from 1986 to 1997.What was the approximate percent increase in consumption from 1986 to 1997 ? A 10% B 20% C 30% D 50% E 80%

20%

Burning one gallon of gasoline in a car releases approximately 20 pounds of CO2 into the atmosphere.One person drives 60,000 miles in a car that averages 30 miles per gallon (mpg), while another person drives 60,000 miles in a car that averages 20 mpg. Over the course of the 60,000 miles, how many fewer pounds of CO2 are released by the 30 mpg car than by the 20 mpg car? A 2,000 B 8,000 C 20,000 D 80,000

20,000

The net annual primary productivity of a particular wetland ecosystem is found to be 8,000 kcal/m2 per year. If respiration by the aquatic producers is 12,000 kcal/m2 per year, what is the gross annual primary productivity for this ecosystem, in kcal/m2 per year? A 4,000 B 8,000 C 12,000 D 20,000 E 96,000

20,000

A temperate grassland biome has a gross primary productivity of 3,480kilocalories/meter2/year3,480kilocalories/meter2/year and a net primary productivity of 2,000kilocalories/meter2/year2,000kilocalories/meter2/year. Which of the following is the approximate number of kilocalories/meter2/yearkilocalories/meter2/year available to herbivores in that biome? A 200kilocalories/meter2/year200kilocalories/meter2/year B 1,480kilocalories/meter2/year1,480kilocalories/meter2/year C 2,000kilocalories/meter2/year2,000kilocalories/meter2/year D 5,480kilocalories/meter2/year

200kilocalories/meter2/year200kilocalories/meter2/year

The Chernobyl nuclear disaster led to the release of massive radiation, specifically iodine-131 and cesium-137, which has been connected to a variety of environmental problems in the 3030 years following the disaster. A soil sample near Chernobyl was found to contain 187kBq/m2 of cesium-137. If the half-life half-life of cesium-137 is approximately 3030 years, how much cesium-137 will remain in the sample after 9090 years? A 93.50kBq/m B 23.38kBq/m2 C 6.23kBq/m2 D 1.58kBq/m2

23.38kBq/m2

Directions: Select the choice that best fits each statement. The following question(s) refer to the histograms below, which show age structure expressed as percentage of population for Countries I, II, and II in 1990. Approximately what percent of the population in Country II is under age 15 A 1% B 5% C 10% D 25% E 50%

25%

Directions: Select the choice that best fits each statement. The following question(s) refer to the histograms below, which show age structure expressed as percentage of population for Countries I, II, and II in 1990.Approximately what percent of the population in Country II is under age 15? A 1% B 5% C 10% D 25% E 50%

25%

Directions: Select the choice that best fits each statement. The following question(s) refer to the graph below which shows the monthly average CO2 concentration, in parts per million, as measured at Mauna Loa Observatory in Hawaii. According to the graph, the increase in CO2 concentration, in parts per million, between 1970 and 1990 is closest to A 5 ppm B 30 ppm C 50 ppm D 340 ppm E 355 ppm

30 ppm

The country of Sudan has an estimated annual growth rate of 2 percent. At this rate of growth, approximately how many years will it take for the population of Sudan to double? A 30 years B 35 years C 50 years D 80 years E 140 years

35 years

The graph below illustrates Arctic sea height from 1993 to 2018. Based on the data in the graph, which of the following represents the approximate rate of change in Arctic sea level from 1995 to 2015 ? A 60mm per year B 10mm per year C 7mm per year D 3mm per year

3mm per year

A country currently has a population of 100 million and an annual growth rate of 3.5 percent. If the growth rate remains constant, what will be the population of this country in 40 years? A 150 million B 200 million C 300 million D 400 million E 800 million

400 million

A pesticide was applied to a population of roaches, and it was determined that the LD50 was 55mgkg. If the average mass of a roach was 0.02kg, which of the following approaches will determine the dose in mg per roach? A 55mg1kg×10.02kg B 55mg1kg×50%×0.02kg C 55mg1kg×0.02kg D 0.02kg1×55mg1kg50%

55mg1kg×0.02kg

If the annual consumption of petroleum in the United States is about 23 barrels per capita, the total annual consumption of petroleum in the United States is closest to A 12 million barrels B 240 million barrels C 2 billion barrels D 6 billion barrels E 10 billion barrels

6 billion barrels

The half-life of a radioactive substance with the decay rate shown in the graph above is closest to A 5 seconds B 7 seconds C 14 seconds D 18 seconds E 21 seconds

7 seconds

Uranium-235 has a half-life of 710 million years. If it is determined that a certain amount of stored U-235 will be considered safe only when its radioactivity has dropped to 0.10 percent of the original level, approximately how much time must the U-235 be stored securely to be safe? A 7.1 × 106 B 7.1 × 107 C 7.1 × 108 D 7.1 × 109 years E 7.1 × 1010 years

7.1 × 10^9 years

In 1997 the World Resources Institute estimated the world's proven oil reserves to be 1,000 billion barrels and the ultimately recoverable reserves to be 2,000 billion barrels. The table below shows the world consumption of oil from 1986 to 1997.At the 1997 rate of consumption, about how long will the estimated 2,000 billion barrels of oil last? A 25 years B 50 years C 75 years D 200 years E 500 years

75 years

An individual has been driving a passenger vehicle to work, averaging 60 miles a week in a car that averages 22 miles per gallon. The individual plans to purchase a hybrid vehicle that averages 50 miles per gallon. If the individual drives to work 50 weeks a year, how much gas will they save if they switch to a hybrid vehicle for their commute? A 28 gallons B 60 gallons C 76 gallons D 107 gallons

76 gallons

The current global human population is about 6.1 billion and is growing at an annual rate of 1.35 percent. If world population were to grow at this rate for the next year, approximately how many people would be added? A 8 × 105 B 8 × 106 C 8 × 107 D 8 × 108 E 8 × 109

8 × 107

The current global human population is about 6.1 billion and is growing at an annual rate of 1.35 percent. If world population were to grow at this rate for the next year, approximately how many people would be added? A 8 × 105 B 8 × 106 C 8 × 107 D 8 × 108 E 8 × 109

8 × 10^7

The graphs above show the population size and distribution of beak sizes in a finch population on the island of Daphne Major in the Galapagos in 1976 and 1978. Smaller-beaked finches are better suited for eating grasses and small seeds. Larger-beaked finches are better suited for cracking larger seeds and nuts. In 1977 a drought took place and the vegetation of the island was affected by the lack of rain. Which of the following was the average beak size of finches on the island of Daphne Major in 1976? A 8.8 mm B 9.5 mm C 10.2 mm D 13.8 mm

9.5 mm

Directions: Select the choice that best fits each statement. The following question(s) refer to the figure below. A, B, C, and D represent population growth curves, E represents the carrying capacity. Represents the biotic potential of the species A B C D E

A

If an area was originally forested and then underwent urban development, which of the following shows the most likely effects on various parts of the water cycle in the area? A B C D E

A

Which of the following is the best example of cogeneration? A A power plant uses both coal and natural gas as fuel. B A trash-to-steam power plant accepts both residential and commercial waste. C Several power plants supply the same city with electrical energy. D Emissions from power plants are captured and stripped of sulfur for commercial sale. E A coal-fired power plant captures waste heat and uses it to heat adjacent buildings.

A coal-fired power plant captures waste heat and uses it to heat adjacent buildings.

Which of the following is the best description of a population that has a stable age distribution? A A large population that is growing at a constant rate B A large population with a negative growth rate C A population that is in the early stages of logistic population growth D A growing population in which the proportions of individuals in the different age classes remain constant E A small population that has not yet achieved exponential growth

A growing population in which the proportions of individuals in the different age classes remain constant

Which of the following statements about levels of biodiversity is correct? A Genetic biodiversity is a measure of the total number of genes in a community. B A narrow distribution of habitats leads to an increase in species diversity. C Global hotspots are areas where species diversity is thriving due to habitat restoration. D A population with high genetic biodiversity is better able to respond to environmental stressor

A population with high genetic biodiversity is better able to respond to environmental stressor

A group of students want to increase the amount of water infiltration around the property of their urban high school by replacing all the existing impermeable walkway pavers with permeable ones. The school board will not move forward with any property changes unless the students conduct an experiment and collect evidence to help prove that their proposed suggestion to increase water infiltration will be effective. The students create two identical pallets using soil taken from the school's property and install impermeable pavers similar to those found on the school's property in one of the pallets. They install permeable pavers in the other pallet. The students pour exactly the same amount of water over both sets of pavers, leave both pallets in exactly the same environmental conditions, and after one hour measure the amount of water that infiltrated the soil. Which of the following identifies and describes the equipment needed to best determine the amount of water that infiltrated the soil? A A scale should be used to measure the mass of both pallets immediately after pouring the water over the pavers and immediately after the one-hour wait time. B A ruler should be used to measure the level of water above the pavers in both pallets immediately after pouring the water over the pavers and immediately after the one-hour wait time. C A soil triangle should be used to determine the soil texture in both pallets before and after the water is added to both pallets. D A turbidity meter should be used to determine the amount of particulate matter in both pallets before and after the water is added to both pallets.

A ruler should be used to measure the level of water above the pavers in both pallets immediately after pouring the water over the pavers and immediately after the one-hour wait time.

Little is known about Africa's elusive aardvarks, but new research says they are vulnerable to climate change like many other species. Hotter temperatures are taking their toll on the aardvark, whose diet of ants and termites is becoming scarcer in some areas because of reduced rainfall. Drought in the Kalahari Desert killed five out of six aardvarks that were being monitored for a year, as well as 11 others in the area. The aardvarks' body temperatures plummeted during the night because they were not getting enough energy from diminished food sources. They tried to conserve energy by looking for insects during the warmer daytime, but their efforts to adapt could not save them. Researchers said some birds, reptiles and other animals use aardvark burrows to escape extreme temperatures, reproduce, and hide from predators. They could have fewer refuges available if aardvark populations shrink because of rising temperatures, they said. The aardvark, which lives in many parts of sub-Saharan Africa, is identified as an animal of "least concern" on an international "red list" of threatened species. There are no indications that the population is changing significantly in southern Africa, though it is speculated that numbers may be declining elsewhere because of habitat destruction, the bushmeat trade and other factors. "Study Finds aardvarks suffering as African climate heats up," Christopher Torchia, Phys.org, July 31, 2017. Which of the following best identifies the author's claim? A Aardvarks are shifting the location of their territories as a response to global climate change. B Aardvarks are a keystone species and their decline could have a trophic cascade effect on the ecosystem. C The loss of aardvarks described in the study was an isolated incident because most aardvark populations are increasing globally. D Providing aardvarks with more burrows will help them maintain their body temperature and increase survival rates.

Aardvarks are a keystone species and their decline could have a trophic cascade effect on the ecosystem.

Denitrification

About featured snippets• Feedback People also ask What is denitrification process? What is denitrification short answer? Denitrification is the process in which anaerobic bacteria convert soil nitrates back into nitrogen gas

The land on a 100-acre farm is equally suited for grazing cattle and growing corn. Of the following ways of distributing land use, which would produce the greatest number of calories of human consumption? A Acres for Grazing Cattle 100 Acres for Growing Corn 0 B Acres for Grazing Cattle 80 Acres for Growing Corn 20 C Acres for Grazing Cattle 50 Acres for Growing Corn 50 D Acres for Grazing Cattle 20 Acres for Growing Corn 80 E Acres for Grazing Cattle 0 Acres for Growing Corn 100

Acres for Grazing Cattle 0 Acres for Growing Corn 100

Which of the following procedures would be best for remediating the effects of soil salinization? A Application of broad-spectrum biocides to kill microorganisms B Application of superphosphate to increase soil fertility C Addition of clay to increase soil water-holding capacity D Addition of large amounts of water to leach out salts E Addition of lime to raise soil pH

Addition of large amounts of water to leach out salts

By the year 2050, world population is expected to approach 10 billion. If the current population trends continue, which region of the world will most likely experience the majority of the growth? A North and Central America B Central and South America C Eastern and Western Europe D Africa and Asia E Australia and New Zealand

Africa and Asia

This area is most susceptible to erosion, leading to increased suspended solids and increased turbidity downstream. A City B Agriculture Fields C Nuclear Power plant D Dam

Agriculture Fields

A farm is experiencing extensive crop loss due to high rains causing overly saturated soils. Which of the following approaches would best help the farmer alleviate the problem? A Fertilizing the area with manure to eliminate the need for inorganic chemical fertilizers B Amending the soil with clay to increase the water-holding capacity of the soil C Amending the soil with sand to increase the permeability of the soil D Using contour plowing to decrease soil erosion

Amending the soil with sand to increase the permeability of the soil

Which of the following best describes a transform boundary? A An area where two tectonic plates move away from each other B An area where two tectonic plates slide past one another C An area where two or more tectonic plates collide D An area where two or more tectonic plates fuse together at a suture zone

An area where two tectonic plates slide past one another

Which of the following best describes the action of an endocrine disruptor? A An endocrine disruptor attacks hormones that invade a body system and cause disease. B An endocrine disruptor blocks the receptor protein binding site of a hormone so that the cell cannot receive a signal. C An endocrine disruptor acts as an organic catalyst, produced by living things to regulate the rate of chemical reactions. D An endocrine disruptor helps to transport substances, such as pesticides, in an organism following exposure.

An endocrine disruptor blocks the receptor protein binding site of a hormone so that the cell cannot receive a signal.

Which of the following is a direct economic advantage associated with aquaculture? A Aquaculture breeds genetically modified fish to increase the amount of biodiversity in the population. B Aquaculture reduces the number of fishing operations in an area. C Aquaculture produces large amounts of seafood for human purchase and consumption year-round. D Aquaculture requires large bodies of water for its operations.

Aquaculture produces large amounts of seafood for human purchase and consumption year-round.

An industry uses natural gas for manufacturing and uses the waste heat to produce electricity. This is an example of which of the following processes? A Cogeneration B Electrolysis C Gas hydration D Gasification E Reclamation

Cogeneration

Estuaries

An estuary is a partially enclosed, coastal water body where freshwater from rivers and streams mixes with salt water from the ocean

An industry uses natural gas for manufacturing and uses the waste heat to produce electricity. This is an example of which of the following processes? A Cogeneration B Electrolysis C Gas hydration D Gasification E Reclamation

Cogeneration

In an apartment building, exhaust from a gas-powered electrical generator is captured to produce hot water to heat a swimming pool. Which of the following is illustrated by this process? A Synergy B Cogeneration C Synfuels D Energy recovery

Cogeneration

The drop in stratospheric ozone levels in the Southern Hemisphere (the "ozone hole") is most evident during which season? A Antarctic spring (October) B Antarctic autumn (April) C Antarctic summer only (January) D Antarctic winter only (July) E Both Antarctic summer and Antarctic winter (January and July)

Antarctic spring (October)

Purity is a major factor in determining the type of coal used for energy. Which of the following types of coal are correctly paired with a description of its purity and common use? A Anthracite is considered the highest grade coal because it is almost pure carbon, and it is often used for domestic heating. B Coking coal is most widely available and contains 60-86% carbon, which makes it ideal for use in coal power plants. C Steam coal has a high carbon content with low amounts of impurities, which makes it ideal for use in the steel industry. D Lignite is the most mature and purest form of coal, and it is often used for large-scale power generation.

Anthracite is considered the highest grade coal because it is almost pure carbon, and it is often used for domestic heating.

Which of the following would be the best location for a wind farm with ten industrial turbines? A Adjacent to a wetland area with a large migratory avian population in the fall and spring B Scattered throughout a residential neighborhood where strong wind gusts are common C Just outside a town near a cave housing a large bat population D Arranged uniformly on a local farm that has fields and pastureland for dairy and grain production

Arranged uniformly on a local farm that has fields and pastureland for dairy and grain production

Directions: Select the choice that best fits each statement. The following question(s) refer to the following processes. (A) Nitrification (B) Denitrification (C) Assimilation (D) Ammonification (E) Nitrogen fixation Plant roots absorb ammonium ions and nitrate ions for use in making molecules such as DNA, amino acids, and proteins. A Nitrification B Denitrification C Assimilation D Ammonification E Nitrogen fixation

Assimilation

Which of the following best explains the process of electricity generation in a hydrogen fuel cell? A At the cathode, water reacts with oxygen to release hydrogen and heat. Electricity is generated as the bonds form between hydrogen and oxygen. The by-product of the reaction is heat. B At the cathode, hydrogen is reduced and then electrons flow to the anode to generate electricity. Oxygen is produced at the cathode. Oxygen and heat are the by-products of the reaction. C At the anode, hydrogen is oxidized and electrons flow in a circuit to the cathode. At the cathode, oxygen reacts with the hydrogen to produce water. Water and heat are by-products of the reaction. D At the anode, hydrogen is oxidized and electrons pass through an electrolyte to the cathode. There, the protons react with water to produce oxygen. Water and heat are by-products of the reaction.

At the anode, hydrogen is oxidized and electrons flow in a circuit to the cathode. At the cathode, oxygen reacts with the hydrogen to produce water. Water and heat are by-products of the reaction.

For the diagram below, assume fracking has not been used with the oil and gas well.Rock layer that can be porous but must have very low to no permeability A B C D

B

For the diagram below, assume fracking has not been used with the oil and gas well.Rock layer that must be porous and have enough permeability for fluid to move through the layer A B C D

B

Directions: Select the choice that best fits each statement. The following questions(s) refer to the following sources of energy.(A) Photovoltaics(B) Geothermal(C) Biomass(D) Hydroelectric(E) Hydrogen fuel More commonly used in the least developed countries than in highly developed countries A Photovoltaics B Geothermal C Biomass D Hydroelectric E Hydrogen fuel

Biomass

Which of the following best describes an advantage of burning biomass rather than burning fossil fuels in developing countries? A Biomass production requires less open space than fossil fuel extraction does, so land is available for other purposes. B Burning biomass is associated with fewer human respiratory concerns than fossil fuels are because it does not give off sulfur oxides and nitrogen oxides. C Biomass production is cheaper than fossil fuel extraction, making it more readily available for use. D Burning biomass is more efficient than burning fossil fuels, providing more available energy from less of the resource.

Biomass production is cheaper than fossil fuel extraction, making it more readily available for use.

Which three sources supply the majority of commercial energy in the world today? A Coal, oil, and natural gas B Solar, wind, and biomass C Nuclear, hydropower, and photovoltaics D Wood, dung, and charcoal E Fuel cells, geothermal, and tidal power

Coal, oil, and natural gas

Which of the following world regions contain the greatest area of rain forest? A Canada and the United States B Eastern and Western Europe C Russia and China D Australia and New Zealand E Brazil and Indonesia

Brazil and Indonesia

The diagram below shows a particular terrestrial food web. Based on the food web shown above, which of the following correctly describes the flow of energy through the trophic levels from producers to quaternary consumers? A Red-tailed hawk, grasshopper mouse, grasshopper, rattlesnake, brittlebrush B Red-tailed hawk, rattlesnake, grasshopper mouse, grasshopper, brittlebrush C Brittlebrush, grasshopper, grasshopper mouse, rattlesnake, red-tailed hawk D Brittlebrush, red-tailed hawk, rattlesnake, grasshopper mouse, grasshopper

Brittlebrush, grasshopper, grasshopper mouse, rattlesnake, red-tailed hawk

Which of the following is the most successful solution to the problem of fish mortality caused by dams? A Placing fish repellents on dam turbines B Eliminating keystone species above the dam C Building fish ladders around the dam D Harvesting the fish before they reach the dam E Dredging the sediments upstream of the dam

Building fish ladders around the dam

Bull trout use a bewildering array of strategies to aid in their survival, from remaining in streams their whole lives, like rainbow trout, to spending part of their lives in the ocean before returning to streams to spawn, just as salmon do. Bull trout are present in only one of two neighboring rivers in the Olympic peninsula, Washington state, and in this one (the Elwha River), where two large dams were removed. Dam removal resulted in massive outflow of sediments, reducing the clarity of the water and also building up a large delta and expanding the size of the estuary at the mouth of the Elwha River. Sampling for bull trout before, during, and after dam removal was used to detect whether bull trout changed their use of the Elwha River estuary or moved into the adjacent Salt Creek stream where they were formerly absent. Sampling revealed no movement into Salt Creek, but numbers of bull trout in the Elwha River estuary increased greatly during and immediately after dam removal, coinciding with large sediment outflow, before returning to their original low levels. Thus, bull trout appear to have used the enlarged estuary as a refuge from the effects of dam removal, then returning to the river when the river water cleared up from the sediment. Of additional interest is the long-term response of bull trout to the additional habitat opened up above the former dams. Lincoln, A. E., J. A. Shaffer, and T. P. Quinn. 2018. Opportunistic use of estuarine habitat by juvenile Bull Trout, Salvelinus confluentus, from the Elwha River before, during, and after dam removal. Environmental Biology of Fishes 101:1559-1569 Which of the following best identifies the author's claim? A Bull trout experienced high mortality rates as a result of the formation of the estuary and sediments released during dam deconstruction. B Bull trout are adaptable to a variety of habitats and were able to survive in the river habitat during the dam deconstruction. C Bull trout remained in the estuary system after the dam deconstruction because the disruption had irreversibly altered the river habitat. D Bull trout are unable to live in high sediment loaded waters and migrated to avoid the conditions introduced during dam deconstruction.

Bull trout are unable to live in high sediment loaded waters and migrated to avoid the conditions introduced during dam deconstruction.

Long-term intensive agriculture that does not add soil amendments such as manure or fertilizer has been shown to decrease soil organic matter content. Which of the following figures best shows this relationship? A B C D E

C

Which soil horizon has very little organic matter and accumulates soluble metals and nutrients that are transported from other soil layers? A B C D

C

A community located downwind from a coal-fired power plant has seen a recent increase in the number of dead and dying trees. A soil scientist measured values for the following parameters before and after the trees died off. Which of the following soil data should be used to determine if the coal-fired power plant emissions were the cause of the damage to the trees? A Moisture content and water retention B Parent material composition C Pesticide and herbicide residue levels D Calcium and aluminum levels

Calcium and aluminum levels

Which of the following describes an environmental problem that can result from the combustion of fossil fuels to generate electricity? A chemical reaction occurs between the fossil fuel and oxygen to produce energy for the generation of electricity. B Carbon dioxide is a product of the combustion fossil fuels, and it can lead to global climate change when released into the atmosphere. C The combustion of fossil fuels can heat water to convert the water into steam to turn turbines that are connected to electrical generators. D Combustion reactions using fossil fuels require more energy than they produce and are therefore very inefficient.

Carbon dioxide is a product of the combustion fossil fuels, and it can lead to global climate change when released into the atmosphere.

Which of the following is true of carbon as it cycles in nature? A Carbon dioxide is released during photosynthesis. B Carbon compounds rarely exist in the gaseous state. C Carbon sinks include forests and oceans. D The carbon dioxide concentration in the atmosphere is reduced by cutting trees. E Carbon is concentrated in igneous rocks.

Carbon sinks include forests and oceans.

This rate of reaction can be measured in the dark by determining the amount of oxygen gas consumed in a period of time. A Primary productivity B Gross primary productivity C Net primary productivity D Cellular respiration

Cellular respiration

Which of the following countries has the largest coal reserves? A Saudi Arabia B China C India D France E Japan

China

Which of the following countries has the largest coal reserves? Which of the following countries has the largest coal reserves? A Saudi Arabia B China C India D France E Japan

China

100 ml of water being poured through a soil sample. After the water has passed through the soil, 98 ml of water is measured in the collection tray below the sample. Of the following, which is the most likely composition of the soil sample? A Clay 80%Silt 10%Sand 10% B Clay 50% Silt 40% Sand 10% C Clay 40%Silt 50%Sand 10% D Clay 30%Silt 40%Sand 30% E Clay 10% Silt 10%Sand 80%

Clay 10% Silt 10%Sand 80%

Which of the following is the correct order of soil particles in order of increasing size? A Clay—sand—silt B Clay—silt—sand C Sand—clay—silt D Sand—silt—clay E Silt—clay—sand

Clay—silt—sand

Which of the following best exemplifies population momentum? A Continued growth of a population after fertility drops to replacement level B Continued growth of a population due to emigration C Decreased population due to increase in the death rate D Decreased population due to a reduced death rate and an increased fertility rate E Growth of a population after the fertility rate doubles

Continued growth of a population after fertility drops to replacement level

At which of the following sites would activity result in volcanoes, island arcs, and mountains? A Divergent B Convergent C Transform D Hot Spot

Convergent

The figure below illustrates how a thermal inversion can form over major urban centers. In the figure above, the letters A, B, and C represent the different thermal layers present in an inversion. Which of the following shows the correct sequence of thermal layers as one moves vertically upwards from the surface of Earth? A Warm sea breeze →→ Cool →→ Warm B Warm sea breeze →→ Warm →→ Cool C Cool sea breeze →→ Warm →→ Cool D Cool sea breeze →→ Cooler →→ Coolest

Cool sea breeze →→ Warm →→ Cool

We Need a New Green Revolution In recent years, farm production has stopped growing in the United States, and agriculture research is no longer a priority; it constitutes only 2 percent of federal research and development spending. And, according to the Department of Agriculture, total agricultural production has slowed significantly since the turn of the century. We need another ambitious surge in agricultural science. Experience has shown that the best way forward is funding research through a competitive process. There is a program in the Agriculture Department that embraces these tenets, the Agriculture and Food Research Initiative, and its research grants show great promise. New, hardier varieties of corn are being developed from tropical species that can better withstand heat, drought and changes to the environment. The probiotics found in fermented products like yogurt are being tested to replace antibiotics used in animal husbandry. And nanotechnology and electrified micro-coatings of water are being applied to some produce, to prevent food poisoning. Government research is even exploring how to double the rate of photosynthesis and eliminate the need for pesticides. The potential is great, but the program has never been fully funded. Despite a $25 million increase in the omnibus budget agreement, the budget of the department's research initiative sits at half of what Congress authorized in 2008 when it created the program. Now more than ever, we need to embrace twenty-first century science, fund it and turn it loose so we can develop better methods of putting food on the table. Our world is changing; the way we grow and produce food needs a much richer diet of scientific ingenuity to keep pace. From the New York Times. © 2016 The New York Times. All rights reserved. Which of the following statements best identifies the author's assumptions in terms of a new green revolution? A The global demand for food has remained constant since the 1940s, and therefore there has been minimal pressure to change agricultural practices. B Most current farming practices are eliminating mechanization and increasing profits. C New advances in agricultural technology will need government regulations in order to become effective for large-scale farming operations. D Current agricultural practices and strategies need to be evaluated in order to meet the demand for increased food production.

Current agricultural practices and strategies need to be evaluated in order to meet the demand for increased food production.

For the diagram below, assume fracking has not been used with the oil and gas well.Rock layer that must be rich in organic matter that was subjected to heat and pressure over time during the formation of oil and natural gas. A B C D

D

Which of the following best describes why DDT is classified as a persistent organic pollutant? A DDT is very stable, and as much as 50% of the original concentration can remain in the soil 15 years after the initial application. B DDT can be used to control diseases, such as malaria and typhus, that are spread by insects. C DDT is detected in food worldwide and the general human population is most commonly exposed to the toxin through food. D DDT has both short-term acute and long-term chronic health effects in both humans and wildlife.

DDT is very stable, and as much as 50% of the original concentration can remain in the soil 15 years after the initial application.

In a human population undergoing the demographic transition, which of the following generally decreases first? A Birth rate B Death rate C Average family size D Life expectancy E Level of education

Death Rate

Based on the diagram, which of the following is an expected consequence of runoff and sewage in an aquatic environment? A Decreased oxygen levels in surface water as a result of a decrease in photosynthetic rates by algae B Increased oxygen levels in bottom water from increased rates of cellular respiration in decomposers C Decreased oxygen production by seaweed on the seafloor from an increase in turbidity D Increased oxygen production by decomposition of dead fish and other aquatic organisms

Decreased oxygen production by seaweed on the seafloor from an increase in turbidity

Directions: Select the choice that best fits each statement. The following question(s) refer to the following processes. (A) Nitrification (B) Denitrification (C) Assimilation (D) Ammonification (E) Nitrogen fixation Nitrate ions and nitrite ions are converted into nitrous oxide gas and nitrogen gas (N2). A Nitrification B Denitrification C Assimilation D Ammonification E Nitrogen fixation

Denitrification

Directions: Select the choice that best fits each statement. The following question(s) refer to the following processes. (A) Photosynthesis (B) Eutrophication (C) Denitrification (D) Decomposition (E) Transpiration The process by which a soil nutrient is reduced and released to the atmosphere as a gas

Denitrification

Directions: Select the choice that best fits each statement. The following question(s) refer to the following processes. (A) Photosynthesis (B) Eutrophication (C) Denitrification (D) Decomposition (E) Transpiration The process by which a soil nutrient is reduced and released to the atmosphere as a gas A Photosynthesis B Eutrophication C Denitrification D Decomposition E Transpiration

Denitrification

Which of the following best illustrates the concept of the tragedy of the commons? A Destruction of landscape by surfacing mining on private land B Selective harvesting of trees by a timber company in a national forest C Legislation of catch limits to avoid depletion of fish stocks in a shared lake D Inadvertent destruction of beneficial species while attempting to control pests E Depletion of an aquifer by regional farmers

Depletion of an aquifer by regional farmers

A new 450-unit housing development will replace several small farms on the outskirts of Fremont. The development will be composed of 1/4-hectare lots with nearly identical footprints, as show below. Storm water runoff from the new development into nearby Samel Creek will likely lead to which of the following? A Biomagnification of mercury in birds of prey such as eagles and osprey B Improved recreational opportunities for swimmers and boaters C Destruction of fish habitats by high levels of suspended solids and sediments D Contamination of drinking water supplies by volatile organic compounds such as ozone E Increased incidence of waterborne diseases such as malaria

Destruction of fish habitats by high levels of suspended solids and sediments

Economic benefits of building large dams include which of the following? I. Storage of water for agriculture and domestic use II. Controlling floods upstream III. Production of renewable energy A I only B II only C I and III only D II and III only E I, II, and III

I and III only

Which of the following components, which are not shown on the diagram, are part of the tertiary treatment of wastewater? A Wet scrubbers B Filters C Coagulants D Disinfectants

Disinfectants

Which of the following is the primary environmental problem associated with the use of nuclear power to generate electricity? A Radon leaking into buildings B Production of greenhouse gases such as carbon dioxide C Disposal of radioactive waste D Depletion of the ozone layer E Production of acid rain

Disposal of radioactive waste

Which of the following methods of agricultural irrigation results in the loss of the least amount of water by evaporation? A Conventional center-pivot irrigation B Drop irrigation C Laser-level irrigation D Flood irrigation E Gravity-flow irrigation

Drop irrigation

Researchers are evaluating the waste treatment facility located on the Moose River in the northeastern United States. They are sampling the river and its tributaries at several locations, as shown on the map below. Researchers suspect that sewage leaked into the Moose River. Rates for which of the following diseases should be used to determine whether the human population using the river as a source of drinking water was exposed to sewage? A Dysentery B Mesothelioma C Asthma D Malaria

Dysentery

Directions: Select the choice that best fits each statement. The following question(s) refer to the figure below. A, B, C, and D represent population growth curves, E represents the carrying capacity. Represents the maximum number of individuals that can be supported by a particular ecosystem on a long-term basis A B C D E

E

The diagram above illustrates how the number of individuals in a population changed with time as a result of external stresses and resource limitations. Which lettered portion of the curve most likely corresponds to the carrying capacity of the ecosystem? A B C D E

E

Earth currently rotates on an axis at an angle of 23.5 degrees. If that axis changed to 20 degrees, so that the axis was closer to vertical, which of the following would be expected to occur? A The number of days in a year would be reduced. B Earth would experience increased volcanic activity. C The amount of energy Earth receives from the Sun would increase. D Earth would experience a change in seasonal timing and length.

Earth would experience a change in seasonal timing and length.

The diagram below shows the tectonic plate activity on the West Coast of the United States. Which of the following statements correctly explains the tectonic plate activity on the West Coast of the United States? A The Cascade Mountains were formed when two continental plates converged. B Earthquakes occur at the San Andreas Fault when stored energy is released from locked plates. C The divergence of two plates has led to the formation of volcanoes in the Cascade Mountains. D Movement of plates along the San Andreas Fault created the Cascade Mountains.

Earthquakes occur at the San Andreas Fault when stored energy is released from locked plates.

Which of the following best explains how environmental stressors, such as wildfires, can affect biodiversity in an ecosystem? A After an environmental stress, a genetic bottleneck may occur, which will increase genetic diversity. B Habitat diversity will increase the available niches if the landscape becomes more uniform after a disturbance. C Ecosystems with more species diversity are more likely to recover after a disturbance than ecosystems with low species diversity. D Smaller populations are less likely to go extinct than larger populations, so the species diversity will remain constant.

Ecosystems with more species diversity are more likely to recover after a disturbance than ecosystems with low species diversity.

Directions: Select the choice that best fits each statement. The following question(s) refer to the data table below that shows the annual anchovy catch and average sea surface temperature off the coast of Peru from 1995 through 2001. The high sea surface temperature in 1998 was most likely due to A global warming B El Niño C eutrophication D salinization E Pacific Annual Oscillation

El Nino

Which of the following is the most valid inference regarding electricity production, based on the graph above? A Electricity production from coal began to decrease in 2008 because of shortages in coal supplies. B Electricity production from coal began to decrease in 2008 because of the increasing use of natural gas and renewable energy. C The use of nuclear energy increased from 1970 to 2010 because of the low cost of constructing nuclear power plants. D The use of nuclear energy increased from 1970 to 2010 because of shortages in coal supplies. E Renewable energy has not increased as much as nuclear energy has since 1970 because of federal laws restricting construction of wind farms.

Electricity production from coal began to decrease in 2008 because of the increasing use of natural gas and renewable energy.

The two pie charts show municipal solid waste (MSW) in the United States, 2015. Based on the data in the graphs, which of the following strategies would best increase the total volume of municipal solid waste being recycled? A Free or low-cost pickup of grass trimmings by municipal waste haulers B Increased use of plastic packaging for consumer products C Enacting a single-stream recycling program and providing bins to residential homes D Digitization of news and use of e-mail for communication

Enacting a single-stream recycling program and providing bins to residential homes

Elements that cycle in the environment and that also have a gaseous phase at some point in their cycle include which of the following? Carbon Phosphorus Sulfur A I only B III only C I and II only D I and III only E I, II, and III

I and III only

As a country goes through the demographic transition, the greatest rate of population growth takes place during which phase? A The preindustrial B The pretransitional C The transitional D The industrial E The postindustrial

The transitional

Region with the lowest atmospheric pressure A Troposphere B Stratosphere C Mesosphere D Thermosphere E Exosphere

Exosphere

Based on the data provided, which of the following descriptions best identifies the types of farms that are being compared? A Farm A is a free-pasture farm for cattle, whereas Farm B is a free-range farm for poultry. B Farm A is a concentrated animal feedlot for beef production, whereas Farm B is a cage-free farm for poultry. C Farm A is a grass-fed pasture for beef production, whereas Farm B is a traditional concentrated animal feedlot for cattle. D Farm A is a cage-free farm for poultry, whereas Farm B is an industrial poultry farm.

Farm A is a grass-fed pasture for beef production, whereas Farm B is a traditional concentrated animal feedlot for cattle.

Researchers are evaluating the waste treatment facility located on the Moose River in the northeastern United States. They are sampling the river and its tributaries at several locations, as shown on the map below. Which of the following water quality tests would best indicate if sewage from the wastewater treatment plant has contaminated the Moose River? A Dissolved oxygen measured at site A B Fecal coliform measured at site B C Nitrates measured at site C D Temperature measured at site D

Fecal coliform measured at site B

Which of the following examples includes both a point and a nonpoint source of pollution? A A factory smokestack and a sewage overflow B Runoff from city streets and pesticides from agricultural fields C Waste from animal feedlots and sediment from construction sites D Fertilizer from suburban lawns and wastewater from a water treatment plant

Fertilizer from suburban lawns and wastewater from a water treatment plant

Based on the diagram, which of the following best describes how a malfunctioning air compressor would affect the process of sewage treatment? A The oxygen concentration in the aeration tank would increase. B The settling of large debris, such as wood, from the water would be prevented. C Fewer bacteria would be available to absorb dissolved organics in the tank. D The sludge pump would return activated sludge to the aeration tank at a slower pace.

Fewer bacteria would be available to absorb dissolved organics in the tank.

Which of the following accurately describes a fuel cell vehicle? A Fuel cell vehicles run on both an internal combustion engine and a backup battery. B Fuel cell vehicles are able to convert waste oil from the food industry into ethanol, which is burned as a source of power. C Fuel cell vehicles use hydrogen to generate electricity that runs the motor. D Fuel cell vehicles generate energy by creating a flow of electrons in a semiconducting material such as silicon. E Fuel cell vehicles use methane to heat water, which creates steam that turns a turbine to generate electricity.

Fuel cell vehicles use hydrogen to generate electricity that runs the motor.

Which of the following can be used to assess the biological diversity of an area? A Population size of each species and area occupied by each population B Minimum population area and minimum viable population size C Ratio of r-strategists to K-strategists and life expectancy of K-strategists D Number of individuals under fifteen years old and number of individuals over sixty-five years old E Genetic variation within each species and number of species present

Genetic variation within each species and number of species present

Which of the following is a disadvantage associated with the genetic modification of crops? A Genetically modified crops have a decreased resistance to drought. B Genetically modified crops have a shorter shelf life and are more difficult to transport long distances. C Genetically modifications can decrease the genetic diversity of crop species. D Genetic modifications decrease nutritional content in foods.

Genetically modifications can decrease the genetic diversity of crop species.

Directions: Select the choice that best fits each statement. The following questions(s) refer to the following sources of energy.(A) Photovoltaics(B) Geothermal(C) Biomass(D) Hydroelectric(E) Hydrogen fuel Drawbacks include noise and the release of corrosive chemicals A Photovoltaics B Geothermal C Biomass D Hydroelectric E Hydrogen fuel

Geothermal

Which of the following energy sources is not derived directly or indirectly from solar energy? A Geothermal B Photovoltaic C Hydroelectric D Biomass E Wind

Geothermal

Which of the following best describes a disadvantage of geothermal energy production? A Geothermal energy production is a significant source of noise pollution. B Geothermal energy implementation in homes and businesses has a high upfront cost, and electricity is needed to operate pumps. C Geothermal energy generation costs remain stable when gas and oil prices increase. D Geothermal energy is weather dependent and can be generated only at certain times of the year.

Geothermal energy implementation in homes and businesses has a high upfront cost, and electricity is needed to operate pumps

Directions: Select the choice that best fits each statement. The following question(s) refer to the histograms below, which show age structure expressed as percentage of population for Countries I, II, and II in 1990. Countries undergoing rapid populations growth include which of the following? A I only B II only C III only D II and III only E I, II, and III

I only

The harvesting of livestock for meat consumption affects the planet in various ways. Approximately 25% of the land area on Earth is used for raising livestock. One-third of all water used in meat production goes toward producing beef, and the demand for beef is projected to increase by 95% by 2050. The graph below shows the average meat consumption per capita from 2010 to 2012 and the forecasted meat consumption for 2022 for five different countries. Which of the following claims is best supported by the data in the graph? A As the demand for chicken increases and the demand for beef decreases, changes in meat production will decrease the demand on global water supplies. B Meat consumption per capita will decrease in India between 2012 and 2022 because individuals in that country have a primarily vegetarian diet. C Global meat consumption per capita will increase in all five countries between 2012 and 2022 as the countries move through the demographic transition and become more developed. D The use of growth hormones and antibiotics will allow for an increased yield in meat production in all five countries by 2022.

Global meat consumption per capita will increase in all five countries between 2012 and 2022 as the countries move through the demographic transition and become more developed.

A person's ecological footprint is the total amount of land required to offset a person's carbon emissions. The table below shows ecological footprint data for five different students. Each value is a measure of the number of hectares per land required.After reviewing personal footprint data, Student 5 decides to take action to lower his or her overall footprint by reducing money spent on shoes and clothing. Land use in which of the following categories will be most affected by this change? A Food B Carbon emissions C Housing D Goods and services

Goods and services

Based on the diagram, which group of organisms would be considered herbivores? A Snakes B Mice C Grasshoppers D Grasses

Grasshoppers

which group of organisms would be considered herbivores? A Snakes B Mice C Grasshoppers D Grasses

Grasshoppers

Green roofs offer many environmental advantages, including the removal of some pollutants from rainwater. The parameters in the table above were measured in water samples collected after rainfall had passed through green roofs and over traditional roofs (N=5)(N=5) in New York City. Mean levels of pHpH and pollutants in precipitation are shown in the table. Which of the following statements is best supported by the data shown in the table? A Passage of rainwater through the green roofs increased the acidity of the water from the increased formation of carbonic acid. B Green roofs reduced levels of ammonium because the vegetation absorbed the pollutant. C Green roofs are effective at removing all pollutants from rainwater because the plants can absorb the toxins from the precipitation. D Green roofs do little to improve rainwater runoff quality as most pollutants are not significantly different from the control roofs.

Green roofs reduced levels of ammonium because the vegetation absorbed the pollutant.

Based on the diagram, which of the following is associated with the primary treatment of sewage? A Grit chamber B Aeration tank C Effluent D Activated sludge

Grit chamber

The total rate of photosynthesis in a given area. A Primary productivity B Gross primary productivity C Net primary productivity D Cellular respiration

Gross primary productivity

Extraction of natural gas by the process of hydraulic fracturing (fracking) is most likely to result in which of the following environmental problems? A Groundwater contamination B Ozone depletion C Eutrophication D Denitrification E Desertification

Groundwater contamination

Which of the following properties is most likely associated with soils that contain more sand-sized particles than soils that contain more clay-sized particles? A High permeability B Low erosion rate C High density D Low organic content

High permeability

Endocrine disruptors directly affect which of the following in an organism? A White blood cells B Hormones C Iron content D Oxygen levels

Hormones

Mercury concentrations were measured in freshwater shrimp populations in two different ponds, one polluted with mercury and one unpolluted, with a similar food web in each pond. Which of the following best identifies the scientific question that would guide this investigation? A How do different species of shrimp excrete mercury from their bodies? B How does the food web in a pond affect biomagnification of toxins? C How much mercury accumulates in the tissues of freshwater shrimp living in a polluted pond? D How much mercury is found in the tissues of shrimp predators in an unpolluted pond?

How much mercury accumulates in the tissues of freshwater shrimp living in a polluted pond?

Living organisms must acquire energy from their environment. Examples of adaptations that help organisms acquire this energy include which of the following? The dark, heat-absorbing coloration of a reptile. The fangs and claws of a lion The light coloration of a peppered moth A I only B II only C I and II only D II and III only E I, II, and III

I and II only

Economic benefits of building large dams include which of the following? A I. Storage of water for agriculture and domestic use B II. Controlling floods upstream C III. Production of renewable energy A I only B II only C I and III only D II and III only E I, II, and III

I and III only

Energy is transferred along food chains from one stage to the next. Which statement best explains how the energy is transferred? A If a primary consumer stores 100kcal100kcal of energy, then a secondary consumer that feeds on the primary consumer will also have 100kcal100kcal of energy available. B If a primary producer stores 10,000kcal10,000kcal of energy, then a tertiary consumer will have 10kcal10kcal of energy available. C If a primary producer stores 100,000kcal100,000kcal of energy, then a secondary consumer will have 10,000kcal10,000kcal of energy available. D If a secondary consumer stores 500kcal500kcal of energy, then a tertiary consumer that feeds on the secondary consumer will have 5,000kcal5,000kcal of energy available.

If a primary producer stores 10,000kcal10,000kcal of energy, then a tertiary consumer will have 10kcal10kcal of energy available.

Scientists have observed that the health of people living in moderately developed regions, which have developed transportation and industry but lack environmental control measures, is more likely to be affected by urban outdoor pollution than by indoor air pollution. Which of the following conclusions can be drawn from the data presented in the graph? A In moderately developed regions such as China and Latin America, an individual's health is more or as likely to be affected by urban outdoor pollution than by indoor pollution. B Developing regions such as sub-Saharan Africa are less affected by indoor air pollution because there is no source of radon in the region. C Indoor air pollutants decrease and urban outdoor pollutants increase as countries become more industrialized. D The amount of both indoor and outdoor air pollution is directly linked to the size of each region.

In moderately developed regions such as China and Latin America, an individual's health is more or as likely to be affected by urban outdoor pollution than by indoor pollution.

Directions: Select the choice that best fits each statement. The following question(s) refer to the graph below which shows the monthly average CO2 concentration, in parts per million, as measured at Mauna Loa Observatory in Hawaii. Which of the following is most likely a significant cause of the general trend indicated by the graph? A Reduction of ozone concentrations in the upper atmosphere B Reduction in size of polar ice caps C Increase in energy output of the Sun D Increase in evaporation of ocean water E Increase in consumption of fossil fuels

Increase in consumption of fossil fuels

Which of the following describes an aspect of urbanization that would most increase the amount of carbon dioxide released into the atmosphere compared to rural and suburban areas? A A greater amount of impermeable surfaces in urban areas that prevents organic material from entering soils B Reliance on mass transportation systems in urban areas, such as buses and trains rather than individual cars and trucks, which increases carbon dioxide release C Decreased square footage of living space, per person, in most housing units in urban areas that leads to an increased carbon footprint D Increased amounts of waste per unit area in urban areas that leads to higher rates of decay and decomposition

Increased amounts of waste per unit area in urban areas that leads to higher rates of decay and decomposition

Which of the following is the most likely consequence of runoff transporting chemicals that are endocrine disruptors to a pond? A Decreased turbidity in the pond B Increased frequency of birth defects in fish populations in the pond C Increased mutualistic relationships between fish and frog populations in the pond D Increased dissolved oxygen in the pond

Increased frequency of birth defects in fish populations in the pond

Which of the following would most likely result from the deforestation of large areas of tropical rain forests caused by slash-and-burn agriculture? A Increased ozone thinning because of the release of CFCs B Decreased acid rain because of the sequestration of SO2 C Increased global temperature because of the release of CO2 D Deceased evaporation because of the loss of the tree canopy

Increased global temperature because of the release of CO2

Which of the following is most likely to be the direct result of lack of genetic diversity in a food crop such as corn? A Decreased kernel size B Decreased potential yield C Decreased dependence on chemical fertilizers D Increased susceptibility on plant disease E Increased resistance to pests

Increased susceptibility on plant disease

The country with a population about three times that of the United States is A Australia B Bangladesh C China D Ethiopia E India

India

Directions: Select the choice that best fits each statement. The following question(s) refer to the graph below, which shows different phases related to the birth and death rates for a typical human population over time. Which of the following is most likely the primary cause of high death rates in phase I? A Loss of breeding-age males due to warfare B Loss of breeding-age females due to disease C Large percentage of elderly individuals in the population D Infant and childhood mortality E General starvation due to famine

Infant and childhood mortality

Which of the following best describes the mechanism of the greenhouse effect in Earth's atmosphere? A Ultraviolet radiation from the Sun is absorbed by ozone gas in the stratosphere. B Gamma radiation from the Sun is absorbed at ground level by dust particles in the atmosphere. C Infrared radiation from Earth's surface is absorbed by gases in the atmosphere. D Cosmic radiation from deep space is absorbed by gases in the atmosphere. E Alpha radiation from the Sun is absorbed by water vapor in the atmosphere.

Infrared radiation from Earth's surface is absorbed by gases in the atmosphere.

Over a three-year period, a city's environmental protection department measured the particulate matter (PM2.5) in a ten-square-mile area around one of the city's coal-fired power plants. The annual mean level of PM2.5 measured was 20.0μgm3 over the three-year study period. The National Ambient Air Quality Standard (NAAQS) for PM2.5 is an annual mean level of 12.0μgm3, averaged over three years. The city instructs the manager of the power plant to propose a plan to aid the city in meeting the NAAQS for PM2.5 in the area around the power plant over the next ten years. Which of the following would be the most effective plan for the manager to propose to the city? A Install catalytic converters at the plant. B Install vapor recovery nozzles at the plant. C Install electrostatic precipitators at the plant. D Burn lignite coal instead of bituminous coal at the plant.

Install electrostatic precipitators at the plant.

Throughout the world, especially the less-developed countries, burning wood or other biomass for heating or cooking in homes is common. One negative environmental impact of this practice is the production of particulates indoors, which can be a health hazard to humans.Which of the following describes the best strategy for mitigating this potential problem? A Installing effective ventilation systems in dwellings B Updating dwellings with well-sealed windows C Planting native plants around the dwelling D Equipping dwellings with rainwater collection systems

Installing effective ventilation systems in dwellings

Which of the following is a way to potentially eliminate one of the greatest threats to the world's mangrove wetlands? A Increase desalination to reduce the effects of prolonged drought in mangroves. B Integrate ecologically sound shrimp aquacultural practices with mangrove management. C Construct dams in mangroves to increase sediment load and decrease pollution. D Employ irrigation methods to address potential desertification of mangrove habitats.

Integrate ecologically sound shrimp aquacultural practices with mangrove management.

Switching from customary large-scale agricultural practices to which of the following is most likely to reduce the exposure of farmworkers and nearby residents to toxic chemicals? A No-till cultivation B Integrated pest management C Contour plowing D Crop dusting E Use of cover crops

Integrated pest management

The two pie charts show municipal solid waste (MSW) in the United States, 2015. Based on the data in the graphs, which of the following would result in the largest reduction in volume of municipal solid waste disposed of in landfills? A Introduce composting programs for yard trimmings and food waste B Increase programs to recycle glass and metals C Promote reusing plastics and wood products D Increase funding to clothing donation programs

Introduce composting programs for yard trimmings and food waste

Five islands, A, B, C, D, and E, differ only in distance from the mainland, area, and species diversity. Which island would be predicted to have the highest species diversity? A Island A Distance from Mainland (kilometers) 50 Area (hectares) 1 x 102 B Island B Distance from Mainland (kilometers) 50 Area (hectares) 1 x 106 C Island C Distance from Mainland (kilometers) 50 Area (hectares)1 x 102 D Island D Distance from Mainland (kilometers)1,000 Area (hectares) 1 x 102 E Island E Distance from Mainland (kilometers) 1,000 Area (hectares)1 x 106

Island B Distance from Mainland 50 (kilometers)Area (hectares) 1 x 106

Which of the following is a true statement about passive solar heating? A It is effective only during the summer months. B It is based in part on the principle of the greenhouse effect. C It is not used to heat commercial buildings. D It is not efficient because it cannot produce high-quality energy. E It produces more pollution than heating with an electric heat pump.

It is based in part on the principle of the greenhouse effect.

Which of the following is a true statement about replacement-level fertility? A It equals the average number of children a woman will give birth to during her child-bearing years. B It equals the annual number of live births per 1,000 people in a population. C It equals the natural increase of a population in one year. D It is found by subtracting the number of emigrants from the number of immigrants in one year. E It is greater in countries with high infant-mortality rates than in countries with low infant-mortality rates.

It is greater in countries with high infant-mortality rates than in countries with low infant-mortality rates.

Which of the following is true of the Ogallala Aquifer? A It underlies the African Serengeti and supplies millions of people with freshwater. B It underlies the Argentine Pampas and is underutilized as a source of water. C It underlies eastern Quebec and is a limited source of freshwater. D It underlies the United States High Plains and is being depleted by overuse. E It underlies central Florida and is contaminated by pesticide and fertilizer runoff from farms.

It underlies the United States High Plains and is being depleted by overuse.

Directions: Each of the questions or incomplete statements below is followed by five suggested answers or completions. Select the one that is best in each case. The following questions refer to the following: The data shown were collected in a laboratory experiment in which the effect of pH on the survival of water fleas (Daphnia pulex) was examined. In each trial, 40 live water fleas were added to a solution with the pH as indicated. After two hours, observations were made to determine the number of fleas remaining alive in the sample. Results are presented in the table and in graphical form above. How would including a control group be useful in this experiment? A It would provide a reference for the effects of random environmental factors. B It would provide a number against which percentages can be computed. C It would provide a value for which the standard pH can be measured. D It would provide a standard number to test for statistical uncertainty. E It would provide an end data point for graphical analysis.

It would provide a reference for the effects of random environmental factors.

Negative environmental impacts associated with large-scale hydroelectric projects have been demonstrated in which of the following areas? A James Bay, Quebec B Three Mile Island, Pennsylvania C Kissimmee River, Florida D Mono Lake, California E Aral Sea, former USSR

James Bay, Quebec

The graph below shows three different survivorship curves. Cycads are gymnosperms that were diverse and abundant during the Mesozoic era. Extant species of cycads share many traits with those from the Mesozoic era. Cycads bear cones, are long-lived with slow growth rates, show late reproductive maturity, and have very few offspring. Based on this description, the cycads are A K-strategists with a Type I survivorship curve. B r-strategists with a Type III survivorship curve. C K-strategists with a Type III survivorship curve. D r-strategists with a Type II survivorship curve.

K-strategists with a Type I survivorship curve.

Which trophic level has the least available energy in kilojoules in this food web? A Phytoplankton B Herbivorous zooplankton C Baleen whale D Killer whale

Killer Whale

The Environmental Protection Agency (EPAEPA) of the United States recommends taking corrective measures to reduce exposure to radon gas. The EPA has set an action level of 4p CiL for radon gas. The graph below shows radon samples taken from seventeen buildings in California. The radon samples were taken from the air inside the buildings, and the uranium concentrations were sampled from the soil under the buildings. Data points from four of the seventeen building locations are labeled A-D. Which of the following conclusions can best be drawn from the data in the graph? A Location AA has the greatest need for remediation because the radon levels in the building are the highest and exceed the EPAEPA action level. B Location BB has the lowest need for remediation because the radon levels in the building are below the suggested action level. C Location CC has the greatest need for remediation because the radon levels in the soil are the highest. D Location DD has the lowest need for remediation because there is no measurable radon in the air or the soil samples.

Location AA has the greatest need for remediation because the radon levels in the building are the highest and exceed the EPA action level.

A team at a university in Pennsylvania has concerns about the health of streams in a watershed because of the various land uses in the area. There has been an increase in road repair work, and the area has many dairy farms. Which of the following would be most effective at preventing sediment runoff into the streams of the watershed? A Have construction crews steepen the slopes of the valleys of streams near roadwork zones. B Maintain zones of grass at least 100m wide on the sides of streams in the area. C Construct at least one dam on a stream in the watershed to improve flood control. D Shift from dairy farming to corn planted in agricultural fields located within 100m of streams.

Maintain zones of grass at least 100m wide on the sides of streams in the area.

The shaded area in New York State, Pennsylvania, and Ohio in the northeast United States is referred to as a snowbelt. On average, this region receives much more snowfall than surrounding areas. Which of the following best explains why the shaded area receives more snowfall than the other parts of New York, Pennsylvania, and Ohio? A Moisture-laden storms from the Gulf of Mexico release precipitation here because of the cool temperatures in the region. B Air pollutants from power plants and industrial emissions in the Midwest increase snowfall rates. C Moisture picked up from the warmer water in the Great Lakes is deposited as snow downwind. D Sea spray from the Atlantic Ocean is carried westward by prevailing winds.

Moisture picked up from the warmer water in the Great Lakes is deposited as snow downwind.

The graph below shows the total fertility rate in 2010 for several countries. Which of the following statements best describes the trends in the data shown in the graph above? A Countries that are larger in total area have higher total fertility rates. B Rwanda and Zambia are most likely experiencing population decline. C More-developed countries tend to have lower total fertility rates than less-developed countries. D By 2010, all countries had reached replacement-level fertility rates.

More-developed countries tend to have lower total fertility rates than less-developed countries.

Many environmentalists consider Egypt's Aswan High Dam on the Nile River to be an ecological disaster. Which of the following is the best evidence to support this viewpoint? A Creation of the dam flooded important archaeological sites. B Sardine populations near the Nile delta increased exponentially as a result of decreased annual rates of water flow. C The Aswan High Dam supplies one-third of the electrical power used in Egypt. D Some countries upstream from the dam have diverted Nile River water for their own projects. E Most of the nutrient-rich silt that the Nile annually deposited downstream is now deposited behind the dam.

Most of the nutrient-rich silt that the Nile annually deposited downstream is now deposited behind the dam.

Noise Exposure and Public Health Exposure to noise constitutes a health risk. There is sufficient scientific evidence that noise exposure can induce hearing impairment, hypertension and heart disease, annoyance, sleep disturbance, and decreased school performance. For other effects such as changes in the immune system and birth defects, the evidence is limited. Most public health impacts of noise were already identified in the 1960s, and noise abatement is less of a scientific but primarily a policy problem. A subject for further research is the [clarification] of the mechanisms underlying noise-induced cardiovascular disorders and the relationship of noise with annoyance and nonacoustical factors modifying health outcomes. A high-priority study subject is the effect of noise on children, including cognitive effects and their reversibility. Noise exposure is on the increase, especially in the general living environment, both in industrialized nations and in developing world regions. This implies that in the twenty-first century noise exposure will still be a major public health problem. Which of the following best describes why the authors claim that reducing noise exposure is currently more of a public policy problem than a scientific problem? A There is sufficient scientific evidence that noise exposure can induce hypertension and heart disease. B The relationship of noise with annoyance that negatively impacts human health is a subject for future research. C Most public health impacts of noise were already identified in the 1960s. D It is unclear whether negative cognitive effects are reversible in humans.

Most public health impacts of noise were already identified in the 1960s.

In reef ecosystems, corals often have microalgae living inside them. The algae supply the corals with nutrients, and the coral give shelter to the algae. Which of the following interactions best describes the relationship between the corals and the algae? A Competition B Predation C Mutualism D Parasitism

Mutualism

Scientists are looking for new locations to generate electricity using geothermal energy. Which of the following locations would be the best choice for a geothermal energy generating plant? A Near tectonic plate boundaries B Near fossil fuel resources C Near the equator D Near warm ocean currents

Near tectonic plate boundaries

The energy available to consumers determined by subtracting the energy used by plants from the total energy transformed by the process of photosynthesis. A Primary productivity B Gross primary productivity C Net primary productivity D Cellular respiration

Net primary productivity

Directions: Select the choice that best fits each statement. The following question(s) refer to the following processes. (A) Nitrification (B) Denitrification (C) Assimilation (D) Ammonification (E) Nitrogen fixation Ammonia is converted to nitrite, then to nitrate. A Nitrification B Denitrification C Assimilation D Ammonification E Nitrogen fixation

Nitrification

Which of the following processes is illustrated by the downward arrows from the atmosphere that show the conversion of nitrogen gas into usable forms available to producers? A Nitrogen assimilation B Nitrogen fixation C Photosynthesis D Weathering

Nitrogen fixation

Why do introduced species often become pests? A They displace native species. B They increase biodiversity. C They do not adapt well to local habitats. D They contribute to habitat fragmentation. E They have low biotic potential.

They displace native species.

The graphs below show the energy output of one kilowatt of photovoltaic (PV) capacity of different PV panel systems over a day (graph on the left) and the annual energy production (graph on the right) of the same systems. Dual-axis tracking means that the PV panels are constantly moving to always face the Sun directly. If the PV panels are tilted in a fixed position, the direction of tilt is indicated. Flat PV panels are not tilted in any direction.Which of the following PV panel systems is least effective at producing energy? A Tracking, dual-axis B No tracking, tilted south C No tracking, tilted west D No tracking, tilted east

No tracking, tilted east

The graphs below show the energy output of one kilowatt of photovoltaic(PV) capacity of different PV panel systems over a day (graph on the left) and the annual energy production (graph on the right) of the same systems. Dual-axis tracking means that the PV panels are constantly moving to always face the Sun directly. If the PV panels are tilted in a fixed position, the direction of tilt is indicated. Flat PV panels are not tilted in any direction.If peak electricity use occurs around 2p.m. in the Los Angeles area and avoiding brownouts is more important to the installers of the PV panels than annual energy output, which of the following PV panel systems would be best to use in the area? A No tracking, tilted south B No tracking, tilted west C No tracking, tilted east D No tracking, flat

No tracking, tilted west

Which of the following is likely to minimize soil erosion? A High-yield crops B Deforestation C Herbicide use D Annual plowing E No-till agriculture

No-till agriculture

Noise Exposure and Public Health Exposure to noise constitutes a health risk. There is sufficient scientific evidence that noise exposure can induce hearing impairment, hypertension and heart disease, annoyance, sleep disturbance, and decreased school performance. For other effects such as changes in the immune system and birth defects, the evidence is limited. Most public health impacts of noise were already identified in the 1960s, and noise abatement is less of a scientific but primarily a policy problem. A subject for further research is the [clarification] of the mechanisms underlying noise-induced cardiovascular disorders and the relationship of noise with annoyance and nonacoustical factors modifying health outcomes. A high-priority study subject is the effect of noise on children, including cognitive effects and their reversibility. Noise exposure is on the increase, especially in the general living environment, both in industrialized nations and in developing world regions. This implies that in the twenty-first century noise exposure will still be a major public health problem. Which of the following best describes why the authors claim that noise exposure will be a major health problem in the twenty-first century?

Noise exposure is on the increase in both industrialized nations and in developing regions

Which of the following is a characteristic of nonrenewable energy sources that distinguishes them from renewable energy sources? A The construction of power plants that use nonrenewable energy sources is typically more expensive than the construction of those that use renewable energy sources. B Nonrenewable energy sources are typically less harmful to wildlife and ecosystems than renewable energy sources are. C Nonrenewable energy sources typically require additional new technology in order to be used than renewable energy sources do. D Nonrenewable energy sources typically take a longer time to replenish than renewable energy sources do.

Nonrenewable energy sources typically take a longer time to replenish than renewable energy sources do.

Directions: Select the choice that best fits each statement. The following question(s) refer to the following energy sources.Biomass Wind Tidal energy Nuclear fission Sunlight The source that is not renewable A Biomass B Wind C Tidal energy D Nuclear fission E Sunlight

Nuclear fission

Directions: Select the choice that best fits each statement. The following question(s) refer to the following energy sources.Biomass Wind Tidal energy Nuclear fission Sunlight The source that produces long-lived hazardous wastes A Biomass B Wind C Tidal energy D Nuclear fission E Sunlight

Nuclear fission

Directions: The following question(s) refer to the graph below showing the percent contribution of major fuel sources that have supplied energy for the United States for the past 150 years. The category labeled "Other" includes nuclear power, hydroelectricity, solar and wind power, and other alternative sources of energy.Nuclear power is a component of the line labeled "Other" in the graph. Why is it impossible for nuclear energy to completely replace oil and gas in the future? A Nuclear power is unacceptable to the public, so they will not allow it. B At the current rate of consumption, there is only enough uranium to last for fifteen more years. C Nuclear power can supply electricity, but it cannot replace other uses of oil and gas. D Nuclear energy is too costly for general use. E Nuclear waste contributes more to global climate change than do oil and gas.

Nuclear power can supply electricity, but it cannot replace other uses of oil and gas.

The Chernobyl nuclear disaster led to the release of massive radiation, specifically iodine-131iodine-131 and cesium-137cesium-137, which has been connected to a variety of environmental problems in the 3030 years following the disaster. Which of the following best describes the process of electricity generation at a nuclear power plant? A Nuclear power is generated through fission, which releases a large amount of heat. The heat is used to generate steam, which turns a turbine that powers a generator. B Nuclear power is generated when photons are converted to a direct current using a semiconducting material such as silicon. An inverter is used to convert the direct-currentdirect-current electricity into an alternating current. C Nuclear power is generated from the thermal energy of Earth, which is a result of radioactive decay. The water underground is turned to steam, which turns a turbine to produce electricity. D Nuclear power is generated from the combustion of mined uranium, which provides enough heat to power a generator. The generator turns a turbine to create electricity.

Nuclear power is generated through fission, which releases a large amount of heat. The heat is used to generate steam, which turns a turbine that powers a generator.

Photosynthesis is the major source of which of the following gases in Earth's atmosphere? A CO2 B H2O (water vapor) C O2 D H2 E N2

O2

Which of the following best describes a benefit of increasing the number of offshore wind farms rather than onshore wind farms? A Offshore wind farms will consume less water during operation than wind farms on land because the turbines require less steam for electricity generation. B Offshore wind farms will decrease the maintenance costs associated with electricity generation because they are cheaper to operate than wind farms on land. C Offshore wind farms will emit less carbon dioxide and carbon monoxide than wind farms on land because they are farther from urban areas. D Offshore winds farms will produce up to three times as much electricity as wind farms on land because of increased wind speeds offshore.

Offshore winds farms will produce up to three times as much electricity as wind farms on land because of increased wind speeds offshore.

Of the following sources, which supplies the most commercial energy in the world today? A Solar B Oil C Biomass D Nuclear E Hydroelectric

Oil

Which of the following is a point source pollutant that would likely affect the ecosystem health of a coral reef? A Small pieces of plastic washed in by local currents B Runoff of oil from nearby urban highway systems C Oil released from the broken hull of a shipwrecked tanker D Excess nutrients introduced by fertilization of farmland

Oil released from the broken hull of a shipwrecked tanker

Which of the following is the best example of a point source pollutant? A Oil tanker spill B Farm field C Golf course D Parking lot

Oil tanker spill

The tropical rain forests are known to have a large variety of animals. One example is the orchid mantis that is highly camouflaged when standing on a certain species of orchid found in tropical environments. Another species, the tropical cockroach, can live anywhere in the rain forest where it can find water and a food source. Which of the following statements best describes why it is predicted that the orchid mantis will be more affected by global climate change than the tropical cockroach will be? A As the climate changes, the number of orchids is likely to increase, leading to more competition for space by the mantis population. B Orchid mantises tend to be advantaged in habitats that remain constant; the rain forest habitat is likely to shift because of global climate change. C Orchid mantises are generalists, so global climate change will give them more diverse habitats to colonize. D Climate change will lead directly to the clear-cutting of the forests, resulting in a smaller habitat for both the mantis and the tropical cockroach.

Orchid mantises tend to be advantaged in habitats that remain constant; the rain forest habitat is likely to shift because of global climate change.

Directions: Select the choice that best fits each statement. The following question(s) refer to the following air pollutants. (A) Sulfur dioxide (B) Lead (C) Ozone (D) Hydrocarbons (E) Particulates Considered harmful in the troposphere but beneficial in the stratosphere A Sulfur dioxide B Lead C Ozone D Hydrocarbons E Particulates

Ozone

A scientist is studying PCBs in tertiary consumers. She wants to determine if PCBs can be found in tertiary consumers even though these compounds were banned in the United States in the 1970s. Which of the following best identifies a testable hypothesis for the study? A PCBs are highly soluble in water, so it is unlikely that the compounds will be found in the liver or kidneys of tertiary consumers because the compounds have been banned. B PCBs are persistent organic compounds that are fat-soluble, so they should be found in the fatty tissues of tertiary consumers even though the compounds have been banned for decades. C PCBs have synergistic effects with other toxins in the environment and are broken down in water and soil, so most tertiary consumers are not directly exposed to them. D PCBs are acutely toxic to most forms of life and quickly kill most organisms upon exposure; therefore, tertiary consumers today would be expected to have high levels of the compound in their tissues.

PCBs are persistent organic compounds that are fat-soluble, so they should be found in the fatty tissues of tertiary consumers even though the compounds have been banned for decades.

Which of the following best explains why top predators can have up to 60 times as much of persistent organic pollutant (POP) as their prey and up to 600 times as much of the pollutant as producers? A POPs accumulate in the body fat of living organisms. POPs increase in concentration as they move up the food chain. B POPs can travel over long distances by wind or water. Predators have greater access to different locations than do producers and therefore have higher concentrations of POPs. C POPs are not carbon-based compounds, so animal digestive systems lack enzymes capable of breaking them down. POPs biomagnify in the fatty tissues of animals but not in plants. D POPs are solids, which only animals can ingest. Therefore, POPs have a greater effect on secondary and tertiary levels of food chains.

POPs accumulate in the body fat of living organisms. POPs increase in concentration as they move up the food chain.

The graphs below show the energy output of one kilowatt of photovoltaic (PV) capacity of different PV panel systems over a day (graph on the left) and the annual energy production (graph on the right) of the same systems. Dual-axis tracking means that the PV panels are constantly moving to always face the Sun directly. If the PV panels are tilted in a fixed position, the direction of tilt is indicated. Flat PV panels are not tilted in any direction.Which of the following statements is best supported by the data shown in the graph? A PVPV panels that cannot track the Sun and are in a fixed position tilted to the east produce the most electricity daily and annually. B Of the PVPV panels that cannot track the Sun, those in a fixed position tilted to the west produce more electricity daily and annually than do those in a fixed position tilted to the south. C PVPV panels that cannot track the Sun and are in a fixed position tilted to the south produce the least electricity daily and annually. D PVPV panels that can track the Sun and are constantly moving to always directly face the Sun produce the most electricity daily and annually.

PV panels that can track the Sun and are constantly moving to always directly face the Sun produce the most electricity daily and annually.

Directions: Select the choice that best fits each statement. The following question(s) refer to the following possible relationships between organisms in an ecosystem. (A) Commensalism (B) Parasitism (C) Mutualism (D) Predation (E) Competition Exemplified by ticks feeding on a deer A Commensalism B Parasitism C Mutualism D Predation E Competition

Parasitism

Which of the following best describes the conditions under which peat is formed? A Partially decayed vegetation is placed under low-pressure, aerobic, acidic conditions. B Partially decayed vegetation is placed under high-pressure, anaerobic, alkaline conditions. C Partially decayed vegetation is placed under high-pressure, anaerobic, acidic conditions. D Partially decayed vegetation is placed under low-pressure, aerobic, alkaline conditions.

Partially decayed vegetation is placed under high-pressure, anaerobic, acidic conditions.

Scientists have been monitoring CO2 in the atmosphere from Mauna Loa in Hawaii since 1958. Which of the following units is used by scientists to measure CO2 in the atmosphere? A Dobson units B Percent saturation C Parts per million D pH

Parts per million

100 ml of water being poured through a soil sample. After the water has passed through the soil, 98 ml of water is measured in the collection tray below the sample. Which of the following properties of the soil sample most influences the flow of water through the sample? A Color B Fertility C Permeability D pH E Salinity

Permeability

Which of the following best describes a way that persistent organic pollutants (POPs) cause harm in the environment? A Persistent organic pollutants release carbon dioxide and methane into the atmosphere when buried for disposal. B Persistent organic pollutants release radioactivity during decay. C Persistent organic pollutants are soluble in fat, so they accumulate in an organisms' fatty tissues. D Persistent organic pollutants have magnetic signatures that confuse birds during migration.

Persistent organic pollutants are soluble in fat, so they accumulate in an organisms' fatty tissues.

The water-holding capacity of soil is LEAST likely to be affected by the addition of which of the following? A Clay B Humus C Manure D Pesticide E Sand

Pesticide

The water-holding capacity of soil is LEAST likely to be affected by the addition of which of the following? A Clay B Humus C Manure D Pesticide E Sand

Pesticide

Directions: Select the choice that best fits each statement. The following question(s) refer to the following processes. (A) Photosynthesis (B) Eutrophication (C) Denitrification (D) Decomposition (E) Transpiration The process in which glucose is synthesized by plants A Photosynthesis B Eutrophication C Denitrification D Decomposition E Transpiration

Photosynthesis

Directions: Select the choice that best fits each statement. The following questions(s) refer to the following sources of energy.(A) Photovoltaics(B) Geothermal(C) Biomass(D) Hydroelectric(E) Hydrogen fuelConverts solar energy directly into electrical energy A Photovoltaics B Geothermal C Biomass D Hydroelectric E Hydrogen fuel

Photovoltaics

Which of the following trophic levels represents a primary producer in an aquatic food chain? A Phytoplankton B Herbivorous zooplankton C Carnivorous zooplankton D Baleen whale

Phytoplankton

A group of farmers want to change their pest control methods to reduce the amount of pollution in a nearby river. The data on the cost and percent pest population reduction for several methods of pest control the farmers could consider are presented in the table below.Which of the following methods should the farmers choose to minimize the cost for efficient pest removal while still reaching their goal of reducing the amount of pollution in the river? A Using physical traps B Applying a single dose of pesticides C Planting genetically modified crops D Introducing a natural predator to feed on the pests

Planting genetically modified crops

A beach community was recently negatively affected by a hurricane. Community leaders are looking for a potential solution to prevent damage from future hurricanes. Below is a list of their concerns as well as how important each issue is to the community.Which of the following solutions would best address the issues of greatest concern in the community? A Construction of a seawall B Planting mangrove forests C Construction of an offshore breakwater D Adding large sand dunes

Planting mangrove forests

Which of the following practices would best help to protect against inland and coastal flooding in urban areas? A Regulating the use of pesticides and fertilizers in agriculture B Preserving wetland and coastal mangrove habitats C Increasing the capacity of the sewer lines so they can handle a larger volume of water D Straightening stream channels to increase stream flow

Preserving wetland and coastal mangrove habitats

Which of the following best describes the flow of energy in most terrestrial and near-surface marine ecosystems? A Decomposers break down dead or decaying organisms to release energy and nutrients to be used by producers. B Chemoautotrophs use inorganic energy sources, such as hydrogen sulfide, to create organic compounds from carbon dioxide. C Apex predators feed on other organisms, which releases heat that can be used by producers and primary consumers. D Producers use energy from the sun to make organic matter, such as sugars, from carbon dioxide and water and are then consumed by organisms higher in the food chain.

Producers use energy from the sun to make organic matter, such as sugars, from carbon dioxide and water and are then consumed by organisms higher in the food chain.

In a typical forest ecosystem, dead trees and fallen trees are most important because of their role in which of the following? A Providing a valuable source of timber B Providing habitats for wildlife C Contributing to soil erosion D Increasing water runoff E Removing carbon dioxide from the air

Providing habitats for wildlife

Which of the following correctly describes the disruption of an ecosystem service by an anthropogenic activity? A Provisioning services are disrupted by increased combustion of fossil fuels, which leads to an increase in air pollution. B Regulating services are disrupted by flash flooding from a hurricane, which decreases available food resources. C Supporting services are disrupted by an earthquake that causes habitat destruction, which displaces plants and animals. D Cultural services are disrupted by a tornado that causes the loss of acres of forest, which reduces the amount of harvestable timber.

Provisioning services are disrupted by increased combustion of fossil fuels, which leads to an increase in air pollution.

Raccoons eat a variety of foods and can live in a variety of habitats, including locations near humans. Tiger salamanders eat an abundance of worms and insects and require wetland habitats so they do not dry out. Which of the following best identifies the two different species? A Raccoons are decomposers, and tiger salamanders are scavengers. B Raccoons are carnivores, and tiger salamanders are herbivores. C Raccoons are generalists, and salamanders are specialists. D Raccoons have a Type II survivorship curve, and tiger salamanders have a Type I survivorship curve.

Raccoons are generalists, and salamanders are specialists.

Carbon dioxide equivalent ( CO2e ) is used to compare contributions to climate change of various greenhouse gases. A study at a farm calculated the carbon dioxide equivalent for different production stages for beef and chicken production. Which of the following statements is best supported by the data in the graph? A Producing food for beef and chicken emits a total of 4.67kg CO2e= per kg of edible meat, released during transport and processing of animal feed. B The carbon dioxide equivalent for farm energy use by chickens is the greatest source of emissions, because the 24 hours of lighting that chickens require to grow uses a lot of electricity. C Raising beef contributes more to greenhouse gases than raising chickens does, primarily as a result of fermentation in cows' digestive tracts, which releases methane gas. D Chicken manure does not emit any greenhouse gases, because chickens have unique symbiotic bacteria in their digestive systems.

Raising beef contributes more to greenhouse gases than raising chickens does, primarily as a result of fermentation in cows' digestive tracts, which releases methane gas.

The Chernobyl nuclear disaster led to the release of massive radiation, specifically iodine-131 and cesium-137, which has been connected to a variety of environmental problems in the 30 years following the disaster. A meltdown in which of the following structures at a nuclear power plant, such as Chernobyl, would most likely lead to the accidental release of radiation? A Cooling tower B Turbine C Generator D Reactor core

Reactor core

Based on the data shown in the graph, which of the following best describes how ecologists could determine when this ecosystem is approaching a climax community? A Determine if the keystone species in the ecosystem is present two years after the volcanic eruption B Measure the biomass of the pioneer species present before and after the volcanic eruption C Record the number and type of producers present every year after the eruption D Calculate the net primary productivity of a specific indicator species every year after the eruption

Record the number and type of producers present every year after the eruption

Which of the following would be categorized as a cultural ecosystem service of forests? A Recreation and scenic areas for tourism B Timber and landscape materials C Mushroom and plant harvests D Soil stabilization and air purification

Recreation and scenic areas for tourism

Which of the following is most typically associated with the transition from a rural to an urbanized society? A Reduced birth rates B Reduced need for sewage-treatment facilities C Increased rates of population growth D Increased air quality in urban areas E Increased stabilization of microclimate in urban areas

Reduced birth rates

A person's ecological footprint is the total amount of land required to offset a person's carbon emissions. The table below shows ecological footprint data for five different students. Each value is a measure of the number of hectares per land required.If Student 2 wanted to change his or her footprint to be more similar to that of Student 1, which of the following actions would he or she choose based on the data given in table 1 ? A Using mass transit options when available B Reducing the amount of meat in his or her diet C Moving into a smaller home or an apartment complex D Upgrading his or her electronic devices, such as laptops and cell phones, more often

Reducing the amount of meat in his or her diet

According to the information in the graph above, which of the following would most effectively slow the loss of arable land? A Increasing the efficiency of desalination processes B Reducing urban development C Increasing the efficiency of erosion control D Using more extensive irrigation systems E Discovering cheaper energy sources

Reducing urban development

An integrated pest-management approach to pest control emphasizes which of the following? A Eradication of the pest population B Reliance on spraying broad-spectrum pesticides C Reduction of crop damage to an economically tolerable level D Use of plant monocultures to simplify spraying E Elimination of the use of second-generation pesticides

Reduction of crop damage to an economically tolerable level

Which of the following explains why some energy sources are renewable? A Renewable energy sources exist in a fixed amount and cannot be easily replaced. B Renewable energy sources can be replenished naturally, at or near the rate of consumption. C Renewable energy sources include natural gas and other fossil fuels because they are made from the remains of ancient organisms. D Renewable energy sources use advanced technologies and have been discovered in recent years, making them newer energy sources.

Renewable energy sources can be replenished naturally, at or near the rate of consumption

Which of the following actions would reduce global greenhouse emissions? A Increasing the use of automobiles B Decreasing the number of nuclear power plants C Replacing coal-burning power plants with wind farms D Converting tropical forests to rice paddies E Switching from hydroelectric power generation to power generation using natural gas as the primary fuel

Replacing coal-burning power plants with wind farms

Which of the following strategies could be used to reduce soil erosion during surface mining of coal? A Increasing the use of open pit mining. B Adding air filtration systems. C Replanting vegetation. D Removing only low-grade ores.

Replanting vegetation.

Which of the following is a way for the government to encourage efficient energy A Requiring higher fuel economy standards for new cars B Implementing government subsidies to keep gasoline prices low C Raising the speed limit from 55 to 70 miles per hour D Limiting the development of public transportation systems E Removing all taxes from gasoline at the fuel pump

Requiring higher fuel economy standards for new cars

Long Island Sound is a body of water off the northeast coast of the United States. Over nine million people live in the small watershed area that feeds into the sound. To determine the effect of the population density on the watershed, scientists measured the oxygen content of bottom waters in the sound. The data from late summer 2013 are displayed in the map below. Which of the following factors would most likely contribute to the conditions observed in the Long Island Sound? A Overfishing B Plastic pollution C Sewage treatment D Restricted circulation

Restricted circulation

This freshwater biome is characterized by fast-flowing water that can originate from underground springs or runoff, which carries sediment and organic material. A Coral reefs B Freshwater wetlands C Open ocean D Rivers

Rivers

Which of the following would be the most likely explanation for the changes shown in Scenario 3 ? A Native species were replanted in the areas, which led to fewer niches. B Roads and electric power lines subdivided the landscape into smaller pieces and decreased the amount of available habitat. C The increased edge-to-interior ratio resulted in habitat fragmentation and the formation of smaller, more manageable parcels of land. D Habitat corridors were built to allow individuals between populations to mate, which helped to prevent inbreeding and reduce the genetic diversity often found in isolated populations.

Roads and electric power lines subdivided the landscape into smaller pieces and decreased the amount of available habitat.

Coal was surface mined from five sites, labeled 1 through 5, that had originally been covered by forest. Each site was planted with a mix of tree seedlings to reclaim the area to forest. Grass was also seeded over each site to prevent initial erosion and to provide forage. Ten years after planting, information was collected at the five sites and is given in the table below. Select the letter of the row that indicates the site with the most successful forest reclamation. A Row A B Row B C Row C D Row D E Row E

Row E

Cholera is an acute infection of the digestive system that is caused by the bacterium Vibrio cholerae. Infected individuals can experience a range of symptoms, including diarrhea, vomiting, muscle cramps, low blood pressure, rapid heart rate, and extreme thirst. If symptoms persist and severe dehydration results, an infected individual can die within hours of symptom onset. The diagram below shows how cholera is transmitted.Based on the diagram, which of the following is the most likely reason that a community with good sanitation could still have individuals who contract cholera? A Disease transmission for cholera is through direct contact with infected individuals and is independent of sanitation conditions. B V. cholerae grow rapidly in clear, moving water that has limited nutrients or wastes. C Waste water treatment facilities that use high levels of chlorine to treat the outgoing municipal water supply have an increased risk of cholera outbreaks. D Runoff from a nearby agricultural field pollutes a reservoir used for drinking water.

Runoff from a nearby agricultural field pollutes a reservoir used for drinking water.

Which of the following is most likely to increase both the nutrient levels and the bacterial content of lake water? A Runoff from a nearby hog farm B Thermal pollution from a nearby power plant C Increased aeration of the lake water D Percolation of the water through soil to groundwater E Acidification of the lake water by acid deposition

Runoff from a nearby hog farm

Directions: Select the choice that best fits each statement. The following question(s) refer to the following. (A) Safe Drinking Water Act (B) Clean Water Act (C) Comprehensive Environmental Response Compensation and Liability Act (CERCLA) (D) Resource Conservation and Recovery Act (E) Toxic Substances Control Act Requires minimum safety standards for community water supplies A Safe Drinking Water Act B Clean Water Act C Comprehensive Environmental Response Compensation and Liability Act (CERCLA) D Resource Conservation and Recovery Act E Toxic Substances Control Act

Safe Drinking Water Act

Salamanders, ospreys, snapping turtles, and beavers all live in aquatic habitats. Salamanders lay their eggs in small freshwater pools and feed on insects. Osprey habitats include rivers, bays, and swamps and the birds feed on fish and other animals in the water or on land. Snapping turtles live in fresh or brackish water and feed on plants, fish, frogs, and snakes. Beavers build lodges on islands, pond banks, and lake shores and eat a variety of shrubs, grasses, and crops. Based on the information provided above, which of the following species would best be classified as a specialist? A Salamanders B Ospreys C Snapping turtles D Beavers

Salamanders

Overuse of groundwater in coastal areas would most likely result in which of the following? A Rise in water table B Increase in stream flow C Bacterial contamination of surface water D Saltwater intrusion E Decrease in eutrophication

Saltwater intrusion

The excessive pumping of groundwater from wells can influence the water levels below ground. Which of the following is more likely to occur in aquifers in coastal areas than in aquifers in other regions? A Upward movement of the water table B Saltwater intrusion into the aquifers C The formation of a cone of depression D An increased rate of recharge from precipitation

Saltwater intrusion into the aquifers

Of the following, which has the greatest permeability? A Clay B Loam C Sand D Silt E Humus

Sand

Based on the soil texture triangle, which of the following would describe soil that was 25% clay, 25% silt, and 50% sand? A Loam B Clay loam C Sandy loam D Sandy clay loam

Sandy clay loam

Corn is a crop that grows best in soil that drains well. Which of the following soils is best suited to grow corn? A Clay because it has at least 45% clay so it absorbs water very well. B Silt loam because it is about 80% silt and has low amounts of sand. C Sandy loam because it has large amounts of sand and less than 20% clay. D Silty clay loam because it has all three soil components.

Sandy loam because it has large amounts of sand and less than 20% clay.

During an El Niño-Southern Oscillation event, which of the following best describes conditions in the eastern part of the tropical Pacific Ocean (e.g., near Peru and Ecuador)? A Sea Surface Temperature Low Rainfall Low B Sea Surface Temperature Low Rainfall High C Sea Surface Temperature High Rainfall Low D Sea Surface Temperature High Rainfall High E Sea Surface Temperature High Rainfall Normal

Sea Surface Temperature High Rainfall High

Which of the following is the best example of a keystone species? A Sea otter B Sea urchin C Spotted owl D Snail darter E Condor

Sea otter

Directions: Select the choice that best fits each statement. The following question(s) refer to the graph below which shows the monthly average CO2 concentration, in parts per million, as measured at Mauna Loa Observatory in Hawaii. Which of the following most directly explains the periodic fluctuations of the curve? A Daily variations in air temperature B Daily variations in sea level C Seasonal variations in photosynthetic activity D Seasonal variations in ocean water temperature E Seasonal variations in human industrial activity

Seasonal variations in photosynthetic activity

Scientists are interested in determining if selenium, from a nearby mine, magnifies in the tissues of fish living in a lake. Which of the following best describes a testable hypothesis for the study? A Tissue from fish in the lake will have a higher level of selenium than tissue from fish in a nearby lake. B Selenium will be at a higher concentration in fish tissue than in the tissue of fish prey. C Fish will have a higher concentration of selenium than mercury in their tissues. D Selenium will be at a higher concentration in acid mine drainage than in fish tissues.

Selenium will be at a higher concentration in fish tissue than in the tissue of fish prey.

For which of the following reasons do small isolated islands have a greater rate of species extinction than larger, less isolated islands? A Small isolated islands are more likely to receive more migrating species. B Small isolated islands provide opportunities for a greater diversity of species. C Because of their size, small isolated islands accumulate more species by chance. D Small isolated islands have a lower availability of resources. E Because they have fewer available niches, small isolated islands are targeted and colonized by species.

Small isolated islands have a lower availability of resources.

Which of the following problems is best addressed with contour plowing? A Eutrophication B Denitrification C The pesticide treadmill D Soil erosion E Soil salinization

Soil erosion

Which of the following best describes why the engineer chose to calculate the permeability of the soil sample in the experimental design in order to evaluate a natural containment system to protect groundwater? A Soils that have high permeability would best help protect groundwater because they would allow for faster recharge of the aquifer. B Soils that have high permeability would best help protect groundwater because they have higher rates of nutrient and pesticide leaching. C Soils that have low permeability would best help protect groundwater because the soil traps pollutants so that chemical and biological breakdown can occur. D Soils that have low permeability would best help protect groundwater because the large pore spaces would provide space for pollutants to attach and would have rapid percolation of water.

Soils that have low permeability would best help protect groundwater because the soil traps pollutants so that chemical and biological breakdown can occur.

A farmer observes that increasingly higher concentrations of a particular pesticide have been required each year over the past ten years to achieve the same level of effectiveness on a specific insect pest. Which of the following best helps explain the observation? A Some individual insects live longer than others and pass on traits acquired during their lives. B Only the most aggressive and territorial insects survive and reproduce. C Some individual insects are more likely than others to survive and reproduce due to their inherited traits. D Some individual insects produce many offspring, and thus their offspring live longer. E Some individual insects reproduce before the pesticide is applied, thereby avoiding its harmful effects.

Some individual insects are more likely than others to survive and reproduce due to their inherited traits.

Which of the following species is most likely to be an invasive species if introduced into a favorable new habitat? A Species Number of Fertilized Eggs Produced per Year Oyster 5,000,000,000 B Species Number of Fertilized Eggs Produced per Year Fish 8,000 C Species Number of Fertilized Eggs Produced per Year Rabbit 12 D Species Number of Fertilized Eggs Produced per Year Bobcat 2

Species Number of Fertilized Eggs Produced per Year Oyster 5,000,000,000

Which of the following examples illustrates the community with the highest biodiversity? A Species Number of individuals A2 B8 C10 D10 E20 B Species Number of individuals A1 B4 C4 D6 E30 C Species Number of individuals A5 B5 C5 D15 E20 D Species Number of individuals A10 B10 C10 D10 E10

Species Number of individuals A10 B10 C10 D10 E10

Which of the following would most likely have the greatest positive impact on the quality of the natural environment worldwide? A Discovery of new reserves of fossil fuel in coastal areas B Increased agricultural production on marginal desert lands C Increased life expectancy in more developed nations D Increased reliance on food from ocean ecosystems E Stabilization or reduction of the size of the human population

Stabilization or reduction of the size of the human population

In a certain country, the birth rate is high, the death rate is high, and there are many children that are part of the workforce. The country is most likely in which of the following stages of the demographic transition? A Stage 1 B Stage 2 C Stage 3 D Stage 4

Stage 1

During which stage of the demographic transition shown above does a population begin to experience an explosive increase in growth? A Stage 1 B Stage 2 C Stage 3 D Stage 4 E After stage 4

Stage 2

Directions: Select the choice that best fits each statement. The following question(s) refer to the following regions of Earth's atmosphere. (A) Thermosphere (B) Exosphere (C) Troposphere (D) Mesosphere (E) Stratosphere The beneficial ozone layer is in this region of the atmosphere. A Thermosphere B Exosphere C Troposphere D Mesosphere E Stratosphere

Stratosphere

Directions: Select the choice that best fits each statement. The following question(s) refer to the following regions of Earth's atmosphere. (A) Thermosphere (B) Exosphere (C) Troposphere (D) Mesosphere (E) Stratosphere The beneficial ozone layer is in this region of the atmosphere. A Thermosphere B Exosphere C Troposphere D Mesosphere E Stratosphere

Stratosphere

This layer of Earth's atmosphere contains the ozone layer that absorbs the sun's ultraviolet rays. A Troposphere B Stratosphere C Mesosphere D Thermosphere

Stratosphere

Directions: Select the choice that best fits each statement. The following question(s) refer to the following energy sources. Biomass Wind Tidal energy Nuclear fission Sunlight The source that is converted directly into electrical energy by photovoltaic cells A Biomass B Wind C Tidal energy D Nuclear fission E Sunlight

Sunlight

Which of the following best describes why understanding a watershed and its boundaries is important in designing housing developments along a river? A The water contained in a watershed is more likely to become polluted if the soil has high permeability, so it is important to locate all point sources to reduce contamination. B Watersheds provide habitats for hundreds of marine species, so it is important to protect their feeding and nesting sites when designing housing for humans. C Watersheds provide space for cultivating land to produce food and other products, so it is important to avoid locating a housing development on the most fertile land. D Surface water and stormwater runoff will drain downstream within the watershed boundaries, so it is important to understand the flow of water through the watershed.

Surface water and stormwater runoff will drain downstream within the watershed boundaries, so it is important to understand the flow of water through the watershed.

Directions: The following questions(s) refer to the following graph. Which tree species grows best with full sunlight? A White oak B Hickory C Dogwood D Sweet gum E Beech

Sweet Gum

Which of the following is most likely to reduce the United States contribution to the increase in concentration of atmospheric CO2? A Requiring all farming practices to be organic B Stopping logging in United States forests C Switching fuel use for residential heating from oil to coal D Switching from cars and trucks to rail for mass transit and freight movement E Preventing depletion of stratospheric ozone

Switching from cars and trucks to rail for mass transit and freight movement

Which of the following is the best method for preventing soil erosion in an agricultural area? A Slash-and-burn agriculture B Integrated pest management C Monocrop farming D Terrace farming

Terrace farming

Which of the following best describes legislation that could be used to address problems with overfishing? A The Convention on International Trade in Endangered Species of Wild Fauna and Flora ( CITES ) uses trade rules to protect commercially valuable species. B The Clean Water Act regulates the discharge of pollutants in surface waters. C The Resource Conservation and Recovery Act ( RCRA ) provides guidelines for the management of aquatic resources. D The Delaney Clause of the Food, Drug, and Cosmetic Act regulates the harvesting of fish species for human consumption.

The Convention on International Trade in Endangered Species of Wild Fauna and Flora ( CITES ) uses trade rules to protect commercially valuable species.

Which of the following statements correctly explains the role of fossil fuels in the production of energy? A The combustion of fossil fuels heats water to produce steam, which then turns a turbine. B The steam released from the combustion of fossil fuels turns a generator, which then powers a turbine. C The heat released from burning fossil fuels rises and turns a turbine, which then releases carbon dioxide. D Steam produced from a turbine ignites the fossil fuels, which then powers the generator.

The combustion of fossil fuels heats water to produce steam, which then turns a turbine.

The increase in the size of Earth's human population in the last century has been dramatic. Which of the following identifies two major contributors to this increase? A New methods of agriculture and the identification of new food crops B The Industrial Revolution and modern medicine C Increased emigration/immigration and decreased warfare D New methods of birth control and decrease warfare E Increased education for women and the development of new pesticides

The Industrial Revolution and modern medicine

Which of the following best exemplifies global collaboration for a sustainable environment? A The Montreal Protocol B The Antarctic Treaty of 1961 C The Clean Air Act of 1972 D CERCLA (Superfund) E NAFTA

The Montreal Protocol

Which of the following is most likely to occur in a forested region that has been recently clear-cut? A The concentration of nitrates in streams running through the region will increase. B The average depth of topsoil will increase. C The water temperature in streams running through the region will decrease. D Volume of runoff after rains will decrease. E The frequency of landslides will decrease.

The concentration of nitrates in streams running through the region will increase.

The axis of Earth in relation to the Sun is tilted at a 23.5 degree slant, as illustrated in the diagram below. Use the diagram below to answer the next three questions. Which of the following correctly describes the season and length of day at the Tropic of Cancer at the time of year shown in the diagram? A The Northern Hemisphere is tilted away from the Sun so it is winter at the Tropic of Cancer and daylight is less than twelve hours per day. B The Northern Hemisphere is tilted away from the Sun, so it is winter at the Tropic of Cancer and daylight is approximately twelve hours per day. C The Southern Hemisphere is tilted toward the Sun, so it is summer at the Tropic of Cancer and daylight is greater than twelve hours per day. D The Southern Hemisphere is tilted away from the Sun so it is summer at the Tropic of Cancer and daylight is approximately twelve hours per day.

The Northern Hemisphere is tilted away from the Sun so it is winter at the Tropic of Cancer and daylight is less than twelve hours per day

Based on the diagram, which of the following describes the most likely effect that glaciers melting in Greenland and the Arctic would have on the surface currents of the North Atlantic? A The East Greenland current will reverse directions as sea levels rise. B The Labrador current will be deflected by icebergs that break off from the glaciers. C The warm surface currents will become cooler, and the cool oceanic currents will become warmer. D The Norwegian current will become cooler and less salty.

The Norwegian current will become cooler and less salty.

Based on the diagram, which of the following changes are most likely to occur in the central and eastern tropical Pacific during El Niño? A The Californian current reverses direction and flows northward. B The Californian current flows southward with increased speed. C The Peruvian current strengthens, leading to cooler-than-average sea-surface temperatures. D The Peruvian current weakens, leading to warmer-than-average sea-surface temperatures.

The Peruvian current weakens, leading to warmer-than-average sea-surface temperatures.

Which of the following is a negative consequence associated with photovoltaic solar cells? A Photovoltaic solar cells increase the reliance on transport of fuels long distances from energy sources to homeowners. B Smog and ground-level ozone can be created from pollutants released from photovoltaic solar cells during electricity generation. C The manufacture of photovoltaic solar cells involves toxic metals that can be released into the environment. D The installation of photovoltaic solar cells on houses and other buildings can result in the release of chemicals that destroy stratospheric ozone.

The manufacture of photovoltaic solar cells involves toxic metals that can be released into the environment.

Which of the following best identifies a key component of the hydrologic cycle that powers the movement of water and is missing from the diagram? A Terrestrial animals B The Sun C Bacteria D Volcanoes

The Sun

The axis of Earth in relation to the Sun is tilted at a 23.5 degree slant, as illustrated in the diagram below. Use the diagram below to answer the next three questions. Which of the following areas receives the most intense solar radiation at the time of year shown in the diagram, and why? A The Antarctic Circle, because it is closest to the Sun. B The Tropic of Capricorn, because this latitude is nearly horizontal to incoming solar radiation. C The equator, because it is the hottest region and therefore receives the most intense solar radiation. D The Tropic of Cancer, because solar radiation spreads out over a wider area at this latitude.

The Tropic of Capricorn, because this latitude is nearly horizontal to incoming solar radiation.

Which statement best describes the approximate global distribution of coal supplies? A Brazil and South Africa together contain 33% of proven reserves. B The United States contains 40% of proven reserves. C Australia, Japan, and France together contain 45% of proven reserves. D The United States, the former Soviet Union, and China together contain 50% of proven reserves. E Germany, Brazil, and India together contain 75% of proven reserves

The United States, the former Soviet Union, and China together contain 50% of proven reserves.

Based on the layers in the diagram, which of the following best explains why a thermal inversion tends to concentrate pollutants in the air layer closest to the Earth's surface? A The air layer that is the second closest to the Earth's surface is less dense than the layer above it. B The inversion layer is warmer than the air layer that is farthest from the Earth's surface. C The air layer closest to the Earth's surface is denser than the layer above it. D The air layer that is farthest from the Earth's surface contains the coolest air.

The air layer closest to the Earth's surface is denser than the layer above it

The majority of the nitrogen on Earth can be found in which of the following reservoirs? A Terrestrial plants B Soil bacteria C The atmosphere D Sedimentary rock

The atmosphere

Scientists are interested in determining the average number of species that have migrated from the mainland to a group of islands, as shown below. Partway through their study, they decided to include an additional island in their study group.Based on the theory of island biogeography, which of the following best explains how the scientists' calculation of average number of species would change with the inclusion of this new island in their study? A The average would increase because more species would have migrated directly to the farther island. B The average would decrease because fewer species would have migrated directly to the farther island. C The average would not change because it is likely that the additional island has about the same number of species that migrated as the original island group. D The average would not change because the island is approximately the same size as those in the original group and therefore just as likely to recruit species from the mainland.

The average would decrease because fewer species would have migrated directly to the farther island.

Cholera is an acute infection of the digestive system that is caused by the bacterium Vibrio cholerae. Infected individuals can experience a range of symptoms, including diarrhea, vomiting, muscle cramps, low blood pressure, rapid heart rate, and extreme thirst. If symptoms persist and severe dehydration results, an infected individual can die within hours of symptom onset. The diagram below shows how cholera is transmitted. Based on the diagram above, which of the following best describes why a population in an area that has poor sanitation can be particularly at risk for a cholera outbreak? A V. cholerae is a parasitic bacterium often found in moving water, such as rivers and streams. B Symptoms of the disease can appear anywhere from 2 hours to 5 days after initial ingestion of contaminated food or water. C The bacterium that causes cholera is spread through ingestion of contaminated water or food. D V. cholerae can survive in feces and aquatic environments for up to 2 months.

The bacterium that causes cholera is spread through ingestion of contaminated water or food.

Directions: Select the choice that best fits each statement. The following question(s) refer to the graph below, which shows different phases related to the birth and death rates for a typical human population over time. The rate of population growth starts to slow down at which point? A The end of phase I B The middle of phase II C The beginning of phase III D The end of phase III E The middle of phase IV

The beginning of phase III

A one-hectare pond is sampled in early September. The sample yields 1 small catfish as well as 17 benthic invertebrates that represent 10 species. If the pond is resampled a year later, which of the following would best indicate that the pond had been adversely affected by adjacent development? A An uncommon species has become more numerous. B An increase in low-tolerance species has occurred. C A decrease in high-tolerance species has occurred. D Phylogenetic diversity has increased. E The biodiversity of the pond has decreased.

The biodiversity of the pond has decreased.

Which of the following best describes the change in the carrying capacity of the human population if growth is limited by conditions set forth by the Malthusian theory? A The carrying capacity will increase, and the human population size will grow exponentially. B The carrying capacity will remain constant, and the human population size will stabilize. C The carrying capacity will remain constant, and the human population size will experience an overshoot. D The carrying capacity will decrease, and the human population size will experience a dieback.

The carrying capacity will decrease, and the human population size will experience a dieback.

Which of the following is the most likely unintended consequence of using pesticides in agriculture? A Pesticides increase the amount of vegetation available for cattle in free range grazing, which causes the cattle to graze over a larger land area. B Pesticides can contaminate water sources near agricultural areas causing an increase in primary productivity that leads to eutrophication. C The continual use of pesticides can cause organisms to build a resistance through genetic mutation that then requires more or stronger pesticides. D The continual use of pesticides can lead to an increase in soil biodiversity from increased soil quality.

The continual use of pesticides can cause organisms to build a resistance through genetic mutation that then requires more or stronger pesticides.

A country has a birth rate of 351,000 individuals and a death rate of 131,000 individuals. The death rate was 371,000 individuals in 1950, 281,000 individuals in 1970, and 211,000 individuals in 1990. Which of the following statements best explains why the country has a declining death rate? A The country is in stage 1 of the demographic transition, and infant mortality is high because of poor health services and high levels of poverty. B The country is in stage 2 of the demographic transition, and life expectancy has increased because of improvements in public health, sanitation, and access to food. C The country is in stage 3 of the demographic transition, and more women are entering the workforce and shifting to having children later in life. D The country is in stage 4 of the demographic transition, and the population growth rate is declining.

The country is in stage 2 of the demographic transition, and life expectancy has increased because of improvements in public health, sanitation, and access to food.

Students want to determine the impact of soil erosion from the development of new housing. They plan to measure the density of a species of algae growing along the bottom of a river that flows through their town. The town is built in a heavily forested area, but a large area of trees along the river was recently clear-cut to allow for the construction of a new housing development, which has caused exposed soil to wash into the river. The river flows north to south through the town and is still forested both up- and downstream of the town. Which of the following is the dependent variable in the study? A The tract of land that was clear-cut for the housing development B A tract of land in a heavily forested area outside of town C The density of a species of algae growing along the bottom of the river D The flow rate of the river from north to south

The density of a species of algae growing along the bottom of the river

Which of the following describes a fundamental characteristic of the green revolution in food resources? A The application of higher levels of organic fertilizers to increase rice production B Deforestation to provide field crops with increased sunlight for photosynthesis C The addition of calorie, fat, and fiber percentage to the information provided on food package labels D The development of new strains of crops with higher yields E The discovery that chlorophyll adds nutritional value to wheat, rice, and sorghum

The development of new strains of crops with higher yields

A certain species of dinosaur laid large numbers of eggs, and the hatchlings had high mortality early in life. But if they survived, the young dinosaurs were able to grow at a rapid pace and disperse into the environment in order to thrive. Based on this information and the graph above, which of the following best characterizes this species of dinosaur? A The dinosaurs were r-strategists and exhibited a Type I survivorship curve. B The dinosaurs were r-strategists and exhibited a Type III survivorship curve C The dinosaurs were K-strategists and exhibited a Type II survivorship curve. D The dinosaurs were K-strategists and exhibited a Type I survivorship curve.

The dinosaurs were r-strategists and exhibited a Type III survivorship curve

Construction of a solar power plant is proposed for a desert area near a school. A student has hypothesized that the shade cast by the solar panels will increase the mortality of creosote bush, a native desert plant in the area. Before construction begins, what experimental data should the student collect in order to eventually test the hypothesis? A The rate of increase in atmospheric CO 2 worldwide (in ppm) that occurs in the 12-month period prior to the start of construction B Average noise levels (in decibels) at a construction site, compared to average noise levels in the desert 20 km away from the construction site C The rate of wind-caused soil erosion (in kg/yr) near solar panels at a similar desert location where creosote bush grows D The direct sunlight received by creosote bush in the desert area (in kWh/m2) during a 12-month period E The changes in monthly precipitation (in mm/yr) at a similar desert location where creosote bush grows

The direct sunlight received by creosote bush in the desert area (in kWh/m2) during a 12-month period

Which of the following best describes soil in many tropical rain forests? A They lack soil horizons. B They are quickly depleted nutrients when the forest is removed. C They are similar to soils in grasslands. D They are well suited for growing a wide variety of crops. E They are deep and well drained

They are quickly depleted nutrients when the forest is removed.

Which of the following best describes how the use of hydrogen fuel negatively impacts the environment? A The harvesting of hydrogen fuel from ocean sediments can result in damage to marine ecosystems and fisheries. B The transport of hydrogen fuel requires pipelines that can negatively impact terrestrial habitats, especially when these pipelines leak. C The combustion of hydrogen fuel releases carbon dioxide, which contributes to ocean acidification and global climate change. D The energy needed to produce the hydrogen fuel may come from fossil fuels, whose use contributes to ocean acidification and global climate change.

The energy needed to produce the hydrogen fuel may come from fossil fuels, whose use contributes to ocean acidification and global climate change.

Which of the following is the best example of an economic approach to reduce overfishing for a fish species with a declining population? A The federal government sets a catch limit (per person or per boat) for the species and imposes a tax on any extra fish that are caught. B A conservation agency creates marine parks where the area is a no-catch zone for the species so that the species can recover and population size can increase. C The state government introduces a subsidy for large fishing companies to help them afford to hire more crew members. D An educational group produces signs and video clips to encourage individuals to buy locally caught seafood and to avoid eating big fish like tuna and shark.

The federal government sets a catch limit (per person or per boat) for the species and imposes a tax on any extra fish that are caught.

Which of the following best describes what is represented by the arrows in the food web? A The photosynthetic rates of producers B The flow of energy C The movement of predators D The decomposition of matter

The flow of energy

A person's ecological footprint is the total amount of land required to offset a person's carbon emissions. The table below shows ecological footprint data for five different students. Each value is a measure of the number of hectares per land required.Which of the following is a limitation of using only the data in the table to complete an ecological footprint analysis of the different students? A The footprint for each individual is based on simple calculations to determine approximate resource use. B The footprint uses a top-down approach to provide a complete picture of resource use for comparison between individuals. C The footprint uses global hectares, rather than hectares, to determine the approximate proportion of Earth's surface needed to support the activities of an individual. D The footprint is a static analysis and does not include all environmental impacts of an individual.

The footprint is a static analysis and does not include all environmental impacts of an individual.

Which of the following describes the most likely change to terrestrial biomes resulting from warmer average global temperatures? A The global distribution of midlatitude biomes, such as grasslands and temperate rain forests, would increase. B Deciduous trees found in temperate seasonal forests would flower later in the year. C The latitude range of tropical rain forests would narrow closer to the equator. D High-latitude biomes, such as the tundra, would experience a decrease in net primary productivity.

The global distribution of midlatitude biomes, such as grasslands and temperate rain forests, would increase.

Scientists at the National Oceanic and Atmospheric Administration (NOAA)(NOAA) are forecasting this summer's Gulf of Mexico hypoxic zone or "dead zone"—an area of low to no oxygen that can kill fish and other marine life—to be approximately 7,8297,829 square miles, or roughly the size of Massachusetts. The annual prediction is based on United States Geological Survey river flow and nutrient data. The 2019 forecast is close to the record size of 8,7768,776 square miles set in 2017 and larger than the five-year average measured size of 5,7705,770 square miles. The annually recurring Gulf of Mexico hypoxic zone is primarily caused by excess nutrient pollution from human activities, such as urbanization and agriculture, occurring throughout the Mississippi River watershed. Once the excess nutrients reach the gulf, they stimulate an overgrowth of algae, which eventually die, then sink and decompose in the water. The resulting low oxygen levels near the bottom are insufficient to support most marine life and have long-term impacts on living marine resources that are unable to leave the area. A major factor contributing to the large dead zone this year is the high amount of spring rainfall in many parts of he Mississippi River watershed, which led to record high river flows and much larger nutrient loading to the Gulf of Mexico. This past May, discharge in the Mississippi and Atchafalaya rivers was about 6767 percent above the long-term average between 1980 and 2018. The latest National Climate Assessment predicts an increase in the frequency of very heavy precipitation events in the Midwest, Great Plains, and Southeast regions, which would impact nutrient input to the northern Gulf of Mexico and the size of the hypoxic zone." News release adapted from the United States Geological Survey Which of the following best identifies the author's claim? A The increased size of the hypoxic zone in the Gulf of Mexico is forecasted to benefit the fishing industry in the United States. B The increased rainfall events in the United States are decreasing the tolerance of fish and other marine species in the Gulf of Mexico because of a decreased concentration of oxygen. C Many marine species are migrating away from the Gulf of Mexico as a result of increased nutrient pollution from increased rainfall events. D The large hypoxic zone in the Gulf of Mexico is directly linked to climate change in the Mississippi River watershed.

The increased rainfall events in the United States are decreasing the tolerance of fish and other marine species in the Gulf of Mexico because of a decreased concentration of oxygen.

Which of the following should be included in a watershed map to best determine the non-point source of a pollutant, such as fertilizer, found in a river? A The total volume of groundwater in the aquifer B The total area and the average yearly precipitation of the watershed C The land use upstream of the river and water quality indicators in streams that run downstream into the river D The total volume of surface water of the river and the land use downstream of the river

The land use upstream of the river and water quality indicators in streams that run downstream into the river

What two main factors would best indicate the quality of life of a country's population? A The total fertility rate and the death rate B The crude birth rate and crude death rate C The birth rate and the infant mortality rate D The replacement-level fertility rate and the total fertility rate E The life expectancy and the infant mortality rate

The life expectancy and the infant mortality rate

Which of the following statements about geothermal energy is accurate? A While geothermal energy can heat buildings directly, it is not used to generate electricity. B The location of geothermal energy sources ranges from short distances below ground to miles below the surface. C Steam from underground sources must be condensed before it can be used aboveground. D In some parts of the world, geothermal sources are used to power steam locomotives.

The location of geothermal energy sources ranges from short distances below ground to miles below the surface.

The diagram below represents steps in the phosphorus cycle.Which of the following statements is best supported by the diagram of the phosphorus cycle? A The largest reservoir of phosphorus is in the atmosphere. B The phosphates absorbed by animal tissue through consumption eventually return to the soil. C Plants take up the phosphate ions primarily from the metamorphic rocks. D Phosphates are not soluble in water

The phosphates absorbed by animal tissue through consumption eventually return to the soil.

The diagram above best illustrates major processes in which of the following biogeochemical cycles? A The carbon cycle B The nitrogen cycle C The phosphorus cycle D The water cycle

The phosphorus cycle

Canada has proposed an action plan to address climate change and air pollution. The action plan will reduce the impact of greenhouse gases and pollutants on the environment and human health. Which of the following claims about the Canadian action plan is best supported by the data in the graph? A The plan encourages the transition from electricity generation from nuclear power to electricity generation from cleaner-burning natural gas. B The plan focuses on increasing regulation on installation and maintenance of catalytic converters in vehicle exhaust systems. C The plan supports phasing out traditional coal power plants and transitioning to sources like hydroelectricity, wind, and solar power. D The plan provides a strategy for reducing the amount of plastic wastes in landfills through the production of long-lasting and durable plastics.

The plan supports phasing out traditional coal power plants and transitioning to sources like hydroelectricity, wind, and solar power.

A 163 square-kilometer (km2) small island is found 2,000km from the mainland. A second, larger, 230,000km2 island is found 1,000km from the mainland. Based on the theory of island biogeography, which of the following statements is most likely true about the small island when compared with the large island? A The rate of immigration is lower for the small island than for the large island. B The small island has niches that are more like the mainland than the large island. C The small island has more available resources than the large island. D The rate of species extinction is lower on the small island than on the large island.

The rate of immigration is lower for the small island than for the large island.

The graphs above show the population size and distribution of beak sizes in a finch population on the island of Daphne Major in the Galapagos in 1976 and 1978. Smaller-beaked finches are better suited for eating grasses and small seeds. Larger-beaked finches are better suited for cracking larger seeds and nuts. In 1977 a drought took place and the vegetation of the island was affected by the lack of rain. Which of the following statements best explains the changes in the size and composition of the population of finches after the drought? A The total number of finches in the population increased, and the smaller-beaked finches were more successful in the drier environment. B The total number of finches in the population increased, and the larger-beaked finches were more successful in the drier environment. C The total number of finches in the population decreased, and the smaller-beaked finches were more successful in the drier environment. D The total number of finches in the population decreased, and the larger-beaked finches were more successful in the drier environment.

The total number of finches in the population decreased, and the larger-beaked finches were more successful in the drier environment.

An environmental engineer wants to examine the permeability of different soils to determine the best natural containment system to protect groundwater from pollutants. The engineer tests four different soil samples, places them into a wet column, and pours 50mL of water over each sample. The engineer records the total amount of water that moves through the column as well as the total amount of time for the water to fully percolate through the soil. The data from the study are shown below. Sample Flow Time Total Volume of Water Recovered Permeability of Soil Sample 12.7 seconds 48.5mL 17.96mL sec 22.2 seconds 27.9mL 12.68mL sec 315.8 seconds 14.7mL 0.93mL sec 420.6 seconds 1.9mL 0.09mL sec Which of the following best describes the independent variable in this research design? A The sample of material chosen as the potential container material is the variable being tested, so it is the independent variable. B The flow time, which is related to the porosity of each sample, is the variable that is measured, so it is the independent variable. C The total volume of water recovered is most likely to be affected by human error and is the variable with the largest range of values, so it is the independent variable. D The permeability of each soil sample, which is determined by dividing the total volume of water recovered by the flow time, is the variable that is calculated, so it is the independent variable.

The sample of material chosen as the potential container material is the variable being tested, so it is the independent variable.

Which of the following does NOT exert a major influence on Earth's overall climate? A The annual amount of solar radiation that reaches Earth's surface B The concentrations of the specific gases in Earth's atmosphere C The distribution of land masses and oceans, in combination with Earth's topography D The daily rotation of Earth on its axis, and its annual revolution around the Sun E The seasonal development of temperature inversions in geographical basins

The seasonal development of temperature inversions in geographical basins

Losses of usable energy between successive trophic levels in an ecosystem are best accounted for by which of the following? A The first law of thermodynamics B The second law of thermodynamics C The law of conservation of matter D The process of ecological succession E Limiting factors in the ecosystem

The second law of thermodynamics

Six white-tailed deer and six sika deer were enclosed in a pasture for observation during an eight-year study in central Texas. White-tailed deer are a native species to central Texas, while sika deer are a nonnative species to central Texas. White-tailed deer feed on flowering plants and the tips of trees and shrubs but do not eat grass. Sika deer feed on flowering plants, the tips of trees and shrubs, and grass. All other grazing animals were kept out of the pasture during the study. The number of sika deer more than doubled after the eight years, while the population of white-tailed deer decreased by 50 percent. Which of the following statements best supports the results from the study? A The sika deer are generalists, while the white-tailed deer are specialists. B The sika deer out competed the white-tailed deer in consuming flowering plants and shrubs. C Parasites infected the sika deer population but did not infect the white-tailed deer population. D Change in the local climate reduced the availability of food resources for the white-tailed deer population.

The sika deer out competed the white-tailed deer in consuming flowering plants and shrubs.

Long Island Sound is a body of water off the northeast coast of the United States. Over nine million people live in the small watershed area that feeds into the sound. To determine the effect of the population density on the watershed, scientists measured the oxygen content of bottom waters in the sound. The data from late summer 2013 are displayed in the map below. Based on the image, which of the following conclusions can best be drawn to describe the conditions in the Long Island Sound? A The sound has high levels of dissolved oxygen as a result of storm water runoff from urban areas. B The sound has high levels of phosphorus and nitrogen from an increase in the number of producers. C The sound has large numbers of phytoplankton blooms that are decomposing and consumed by respiring bacteria. D The sound has high numbers of respiring fish populations and very low turbidity.

The sound has large numbers of phytoplankton blooms that are decomposing and consumed by respiring bacteria.

Which of the following factors is fundamentally responsible for seasons on Earth? A The varying distance of Earth from the Sun B The tilt of Earth's axis of rotation C The latitudinal variation in temperature and precipitation D The tidal pull of the Sun and Moon E The anthropogenic greenhouse effect

The tilt of Earth's axis of rotation

Which of the following is a true statement about the total fertility of a society? A The total fertility of a society is the difference between the crude birth rate and the crude death rate. B The total fertility of a society is the number of children necessary for a couple to replace themselves in the next generation. C The total fertility of a society is positively correlated with the average education of women. D The total fertility of a society is negatively correlated with the number of women of child-bearing age. E The total fertility of a society decreases as the society progresses through the demographic transition.

The total fertility of a society decreases as the society progresses through the demographic transition.

Although nuclear energy is considered a cleaner energy source than fossil fuels, some scientists argue this does not support the claim that nuclear energy is renewable. Which of the following claims best supports the argument that nuclear energy is a nonrenewable energy source? A Nuclear energy has low carbon emissions and does not contribute to global warming. B The uranium deposits on Earth are finite. C Nuclear energy is generated by the process of nuclear fission. D Nuclear power plants convert water to steam, similar to fossil fuel-burning power plants.

The uranium deposits on Earth are finite.

Which of the following best describes the method of vehicle transport that is the most environmentally sustainable? A The use of a vehicle powered by hydrogen fuel cells because there are no carbon emissions B The use of a plug-in hybrid vehicle because the fuel sources do not release any carbon dioxide C The use of a vehicle powered by biodiesel because it can be used in conjunction with traditional diesel D The use of a vehicle powered by ethanol because corn is grown to produce energy rather than for human consumption

The use of a vehicle powered by hydrogen fuel cells because there are no carbon emissions B

During an El Niño event, warm surface water moves from the western equatorial Pacific Ocean to the eastern equatorial region. Which of the following best describes how the warm water will affect upwelling off the coast of equatorial South America? A The warm surface water will not change upwelling because this region is at the equator and always has warm water. B The warm surface water will increase upwelling because of the difference in salinity between the warm surface water and the cold deep water. C The warm surface water will decrease upwelling because the cooler temperatures on land in the region will prevent upwelling. D The warm surface water will decrease upwelling because of the density difference between the warm surface water and the cold deep water.

The warm surface water will decrease upwelling because of the density difference between the warm surface water and the cold deep water.

Based on the theory of island biogeography, which of the following is the most likely reason that an island in the ocean that is 5 kilometers from a continent would have a higher number of species than one that is 15 kilometers from a continent? A There are more species that have the ability to travel 5 kilometers than can travel 15 kilometers. B Islands that are closer to continents always have a wider range of habitats than more distant islands do. C There are smaller populations of each species on islands that are at greater distances from continents. D Islands that are closer to continents always have more ecological niches than more distant islands do.

There are more species that have the ability to travel 5 kilometers than can travel 15 kilometers.

Which of the following is a drawback to using biofuels, such as ethanol, as a fuel source? A Ethanol-blended gasoline has higher carbon emissions than petroleum alone. B The global demand for biofuels is continually decreasing. C There is high energy investment in producing and processing the crops needed for ethanol production. D Ethanol-blended gasoline increases the likelihood of freezing gas lines in winter months.

There is high energy investment in producing and processing the crops needed for ethanol production.

Noise Exposure and Public Health Exposure to noise constitutes a health risk. There is sufficient scientific evidence that noise exposure can induce hearing impairment, hypertension and heart disease, annoyance, sleep disturbance, and decreased school performance. For other effects such as changes in the immune system and birth defects, the evidence is limited. Most public health impacts of noise were already identified in the 1960s, and noise abatement is less of a scientific but primarily a policy problem. A subject for further research is the [clarification] of the mechanisms underlying noise-induced cardiovascular disorders and the relationship of noise with annoyance and nonacoustical factors modifying health outcomes. A high-priority study subject is the effect of noise on children, including cognitive effects and their reversibility. Noise exposure is on the increase, especially in the general living environment, both in industrialized nations and in developing world regions. This implies that in the twenty-first century noise exposure will still be a major public health problem. Which of the following best describes why the authors do not claim that noise exposure induces birth defects in humans? A There is sufficient evidence to support that noise exposure negatively impacts the immune system. B Further research is needed concerning the mechanisms underlying noise-induced cardiovascular disorders. C There is insufficient evidence to support that noise exposure induces sleep disturbance in humans. D Noise exposure is on the increase in both industrialized nations and in developing regions.

There is insufficient evidence to support that noise exposure induces birth defects in humans.

Which of the following is considered an advantage of using nuclear power to generate electricity? A No waste is generated. B There is a limitless supply of the raw material needed for the process of generating energy. C Nuclear power plants are inexpensive to build and maintain. D There is no potential for thermal pollution. E There is no CO2 emitted from the nuclear reaction.

There is no CO2 emitted from the nuclear reaction.

Farmers in a suburban coastal town started using no-till agriculture, a practice that reduces soil erosion from their farmland. Use the image above to determine which of the following long-term effects no-till agriculture will have on the ecosystem surrounding the farmland. A There will be an increased need for fertilizer on farmlands. B There will be less algal growth in the nearby ocean. C There will be less phosphorous pollution in the atmosphere. D There will be an increased deposition of phosphorous-rich sediments.

There will be less algal growth in the nearby ocean.

Which of the following is a true statement concerning the production of electricity in conventional nuclear power plants using fission reactors? A New nuclear power plants will be built without containment structures, due to the increased insulation in the reactor core. B Thermal energy is converted into mechanical energy and then to electrical energy, as in coal-burning power plants. C Regularly scheduled releases of radioactive gases during production are well below the maximum contamination levels set by the EPA (Environmental Protection Agency). D Nuclear production of electricity is much less expensive per kilowatt-hour than production of electricity at a coal-burning or natural- gas-fueled power plant. E Storage of nuclear waste is no longer an issue, because power plants are now storing all wastes on-site in specialized containment units.

Thermal energy is converted into mechanical energy and then to electrical energy, as in coal-burning power plants

Which of the following is generally true of K-strategist species as compared to r-strategist species? A They reach sexual maturity earlier. B They have more young. C They are more likely to be invasive species. D They have longer life spans. E Their population cycles are more rapid.

They have longer life spans.

Which of the following best explains why the maximum sustainable yield for ocean fisheries has been exceeded? A Populations of fish-eating birds such as the albatross have increased. B Too many fish of reproductive age are harvested. C Too many marine fish farms have been created. D Everything trapped by large bottom trawl nets is used for food. E For every calorie of fish caught, a ship uses only about 0.5 calorie of fuel energy.

Too many fish of reproductive age are harvested.

El Niño, a periodic warming of ocean surface waters, occurs in which of the following regions? A Tropical East Pacific B Gulf of Mexico C Arctic North Pacific D Temperate West Atlantic E Tropical Indian Ocean

Tropical East Pacific

El Niño, a periodic warming of ocean surface waters, occurs in which of the following regions? A Tropical East Pacific B Gulf of Mexico C Arctic North Pacific D Temperate West Atlantic E Tropical Indian Ocean

Tropical East Pacific

Directions: Select the choice that best fits each statement. The following question(s) refer to the following regions of Earth's atmosphere. (A) Thermosphere (B) Exosphere (C) Troposphere (D) Mesosphere (E) Stratosphere Most oxygen is found in this layer of the atmosphere. A Thermosphere B Exosphere C Troposphere D Mesosphere E Stratosphere

Troposphere

Directions: Select the choice that best fits each statement. The following question(s) refer to the following regions of Earth's atmosphere. (A) Thermosphere (B) Exosphere (C) Troposphere (D) Mesosphere (E) Stratosphere The phenomenon causing global warming occurs primarily in this region of the atmosphere. A Thermosphere B Exosphere C Troposphere D Mesosphere E Stratosphere

Troposphere

Directions: Select the choice that best fits each statement. The following question(s) refer to the following regions of Earth's atmosphere. (A) Thermosphere (B) Exosphere (C) Troposphere (D) Mesosphere (E) Stratosphere The phenomenon causing global warming occurs primarily in this region of the atmosphere. A Thermosphere B Exosphere C Troposphere D Mesosphere E Stratosphere

Troposphere

In which of the following atmospheric layers does most weather occur? A Troposphere B Stratosphere C Mesosphere D Thermosphere

Troposphere

This layer of Earth's atmosphere has the highest density of any atmospheric layer and contains most of the water vapor in the atmosphere. A Troposphere B Stratosphere C Mesosphere D Thermosphere

Troposphere

Directions: Select the choice that best fits each statement. The following question(s) refer to the qualities of water samples. Acidity Turbidity Hardness Dissolved oxygen Salinity Caused by suspended particulates A Acidity B Turbidity C Hardness D Dissolved oxygen E Salinity

Turbidity

Students want to determine the impact of soil erosion from the development of new housing. They plan to measure the density of a species of algae growing along the bottom of a river that flows through their town. The town is built in a heavily forested area, but a large area of trees along the river was recently clear-cut to allow for the construction of a new housing development, which has caused exposed soil to wash into the river. The river flows north to south through the town and is still forested both up- and downstream of the town. Which of the following explains the variable that would be the best for students to measure to determine the direct impact of soil erosion caused by the construction project on the river? A Water temperature, because a decrease in shading in the area would indicate an increase in soil erosion. B Nutrient levels, because a decrease in nitrogen and phosphorus would indicate an increase in soil erosion. C The pH level, because an increase in pH would indicate an increase in soil erosion. D Turbidity, because a decrease in water clarity would indicate an increase in soil erosion.

Turbidity, because a decrease in water clarity would indicate an increase in soil erosion.

Which of the following is the most practical solution to maximizing energy production at wind farms while minimizing the risk to wildlife? A Turn on turbines only at night when most birds and bats are not flying. B Turn off turbines during periods of maximum migration. C Turn on turbines only when winds are below 5mph D Limit the height of turbines to 1010 meters and the width of the blades to 55 meters.

Turn off turbines during periods of maximum migration.

Which of the following best describes an example of resource partitioning in an ecosystem? A Two different bird species feed from the same oak tree; one eats acorns, and the other eats insects in the bark. B A tick uses a white-tailed deer as its host, consuming the deer's blood as food. C Two individuals from the same population of barn owls hunt for the same species of mouse during the night. D A mason bee pollinates a variety of flowers in a botanical garden.

Two different bird species feed from the same oak tree; one eats acorns, and the other eats insects in the bark.

Which of the following best helps to explain why phosphorus is often a limiting factor in many ecosystems? A There is usually a gaseous phase in the phosphorus cycle. B Phosphorus cycles very quickly through environments. C Under many conditions, phosphorus forms stable insoluble compounds. D Phosphate is not readily taken up by plants. E There are no anthropogenic sources of phosphorus.

Under many conditions, phosphorus forms stable insoluble compounds.

In 1950, a country had a total fertility rate of 5.9. In 2010, the total fertility rate of the country was 2.3. Which of the following is most likely the reason for the change in total fertility rate? A Women had increased educational opportunities and had children later in life. B Families had limited access to family planning and contraceptives. C There were lower infant mortality rates as a result of increased access to healthcare. D The country shifted to a more agricultural-based economy and the need for child labor decreased.

Women had increased educational opportunities and had children later in life.

Which of the following best explains the cause of the trend shown in the graph for lead emissions in the United States from 1970 to 2011 ? A By 1990, all new model cars were equipped with internal fabric filters that effectively filtered car exhaust and reduced lead emissions. B Beginning in 1975, The Clean Air Act required new cars to have catalytic converters, which effectively converted the lead found in gasoline to water vapor. C New air-scrubber technologies were introduced in the late 1970s that effectively removed lead in the air and converted it into water vapor. D Unleaded gasoline was first introduced in 1974, and amendments to the Clean Air Act required cars with the model year 1975 or later to use only unleaded gasoline.

Unleaded gasoline was first introduced in 1974, and amendments to the Clean Air Act required cars with the model year 1975 or later to use only unleaded gasoline.

Which of the following metals is considered an energy resource? A Uranium B Cobalt C Mercury D Copper E Palladium

Uranium

Which of the following would be most likely to be used as a fuel source for electricity generation in the plant displayed in the diagram? A Uranium-235 B Cesium-137 C Iodine-123 D Yttrium-90 and yttrium- 91

Uranium-235

Scientists have observed that the health of people living in moderately developed regions, which have developed transportation and industry but lack environmental control measures, is more likely to be affected by urban outdoor pollution than by indoor air pollution. Which of the following activities would most likely account for the levels of indoor air pollution in India and sub-Saharan Africa, as shown in the graph? A Construction of homes with more efficient insulation and increased ventilation B Use of biomass fuel sources such as wood and charcoal for cooking C Industrial power plants burning fossil fuels for electrical energy generation D Use of chemical pesticides on croplands

Use of biomass fuel sources such as wood and charcoal for cooking

Which of the following agricultural practices would most likely lead to improved water quality in nearby streams and rivers? A Using crop rotation in fields B Using nitrogen and phosphorous fertilizers C Converting crop fields to cattle pasture D Clear-cutting areas around crop fields

Using crop rotation in fields

Which of the following solutions would help decrease flood frequencies and flood sizes that are caused by urbanization? A Using new, more permeable materials for roads and sidewalks B Filling nearby wetlands to reduce the amount of water in an area C Encouraging construction of housing developments outward into nearby suburban and rural areas D Installing new, more energy-efficient appliances in existing apartment buildings

Using new, more permeable materials for roads and sidewalks

Cholera is an acute infection of the digestive system that is caused by the bacterium Vibrio cholerae. Infected individuals can experience a range of symptoms, including diarrhea, vomiting, muscle cramps, low blood pressure, rapid heart rate, and extreme thirst. If symptoms persist and severe dehydration results, an infected individual can die within hours of symptom onset. The diagram below shows how cholera is transmitted. Based on the diagram above, which of the following is the most likely reason that V. cholerae can spread rapidly through a population? A V. cholerae is a microscopic bacterium that can survive in various aquatic environments for prolonged periods of time and can cause a delayed onset of visible symptoms. B V. cholerae is a macroscopic bacterium that is easily detected in aquatic environments and causes rapid onset of symptoms. C V. cholerae eventually becomes airborne and is spread through the inhalation of spores, causing a delayed onset of visible symptoms. D V. cholerae is transmitted via an insect vector and often goes undiagnosed in isolated communities.

V. cholerae is a microscopic bacterium that can survive in various aquatic environments for prolonged periods of time and can cause a delayed onset of visible symptoms.

Which of the following sources should be examined in a study to determine the impact of a geologic source on levels of CO2 in the atmosphere? A Photosynthesis B Cellular respiration C Decomposition D Volcanoes

Volcanoes

Which of the following best describes a terrestrial ecosystem that will have the highest net primary productivity? A Warm temperatures, high rainfall, and consistent sunlight B Hot temperatures, low rainfall, and consistent sunlight C Cold temperatures, high rainfall, and inconsistent sunlight D Warm temperatures, low rainfall, and inconsistent sunlight

Warm temperatures, high rainfall, and consistent sunlight

Brown pelicans are fish-eating birds. As a result of an El Niño event, the population of brown pelicans in southern California declines. Which of the following best explains the relationship between the El Niño event and the population decline? A Warmer ocean temperatures can cause fish to move to colder water farther from pelican nest sites. B Lower sea surface heights can result in pelican nest sites being isolated too far from their food supply in the ocean. C The female reproductive tract of birds does not function at the lower water temperatures of El Niño. D Male pelicans get disoriented due to the lack of moisture and fail to return to the nest with food. E Predatory seal populations increase dramatically due to decreased rainfall.

Warmer ocean temperatures can cause fish to move to colder water farther from pelican nest sites

The diagram below shows a model of the global movement of water in the hydrologic cycle.Based on the diagram, which of the following statements about the hydrologic cycle is true? A Water from the ocean evaporates and condenses in the atmosphere. B Surface runoff is the primary recharge of groundwater. C Infiltration of water into the ground prevents water from returning to the oceanic reservoir. D Evapotranspiration from plants decreases atmospheric water vapor.

Water from the ocean evaporates and condenses in the atmosphere.

Which of the following gases is released from the structure labeled C in the diagram? A Carbon dioxide B Methane C Water vapor D Radon-222

Water vapor

Which of the following is a reason why hybrid electric vehicles use less fossil fuel per mile traveled than a typical internal combustion engine vehicle uses? A Hybrid electric vehicles don't burn fossil fuels; they run on electric energy. B Hybrid electric vehicles are powered by the Sun using rooftop photovoltaic panels. C When the brakes are applied in hybrid electric vehicles, kinetic energy is converted into electric energy to charge the batteries that assist the electric motor. D Batteries in hybrid electric vehicles can be charged in the driveway overnight, when rates for electricity usage are typically low. E The engines in hybrid electric vehicles are powered by biofuels made from genetically modified corn.

When the brakes are applied in hybrid electric vehicles, kinetic energy is converted into electric energy to charge the batteries that assist the electric motor.

The two pie charts show municipal solid waste (MSWMSW) in the United States, 2015. Incineration is one solution to reduce the volume of municipal solid waste. Municipal solid waste can be used to produce energy at waste-to-energy plants. Based on the data in the graphs, which of the following materials would be best suited for incineration to reduce total volume, produce energy, and have minimal release of air pollutants? A Glass B Wood C Metals D Plastics

Wood

The supply of Earth's fresh water is fixed and is being depleted at a rapid rate. Which of the following best describes the effects of a limited freshwater supply on human populations? A Disease transmission will decrease as access to water decreases, leading to improved public health. B Conservation measures will increase per capita water consumption, leading to an improved global economy. C Desalinization facilities will be used to process water stored in glaciers and snow packs, leading to new sources of drinking water. D World food production will decrease, leading to increased famine.

World food production will decrease, leading to increased famine.

A large forested area is fragmented into small forest tracts separated by agricultural areas. This change will most likely lead to A an increase in the population of top carnivores B an improvement in the dispersal mechanisms of forest species C a more stable regional climate D a decrease in the amount of edge habitat E a decrease in the gene flow within species of the original forest

a decrease in the gene flow within species of the original forest

Compared with people in developing countries, people in industrialized countries are more likely to eat A beans B beef C corn D rice E wheat

beef

All of the following are negative externalities EXCEPT A illnesses related to living near a manufacturing facility B noise associated with living near a major airport C air pollution caused by refinery emissions D bees pollinating vegetables in addition to creating honey E damage to crops from automobile exhaust

bees pollinating vegetables in addition to creating honey

All of the following substances are derived from petroleum EXCEPT A asphalt B DDT C cellulose D polystyrene E nylon

cellulose

All of the following substances are derived from petroleum EXCEPT A asphalt B DDT C cellulose D polystyrene E nylon

cellulose

An advantage of using natural gas, rather than oil, as a fuel is that natural gas is A less of a contributor to global warming because it does not release CO2 when it burns B less expensive because most reserves are in the United States C more abundant because it is a by-product of photosynthesis D cleaner because it burns more completely E safer to store because it is a gas

cleaner because it burns more completely

The graph below shows the results obtained when two species of Paramecium were grown together in the same medium. A the demographic transition B sustained logarithmic growth C the edge effect D competitive exclusion E the normal distribution

competitive exclusion

The primary cause of Earth's seasons is the A constant tilt of Earth's rotational axis with respect to the plane of its orbit around the Sun B changing distance of Earth from the Sun at different times of the year C periodic wobbling of Earth on its axis of rotation D changing relative positions of Earth, its Moon, and the Sun E periodic changes in solar energy output

constant tilt of Earth's rotational axis with respect to the plane of its orbit around the Sun

The primary cause of Earth's seasons is the A constant tilt of Earth's rotational axis with respect to the plane of its orbit around the Sun B changing distance of Earth from the Sun at different times of the year C periodic wobbling of Earth on its axis of rotation D changing relative positions of Earth, its Moon, and the Sun E periodic changes in solar energy output

constant tilt of Earth's rotational axis with respect to the plane of its orbit around the Sun

Hydraulic fracturing, or fracking, is the pumping of highly pressurized water with a mixture of sand and chemicals into boreholes to A create cracks within the rock in order to provide pathways for the release of gas and oil B pulverize the rock in order to let gas and oil rise through the soil horizons C reopen old mine shafts, from which gas and oil can be collected D create seismic movement and artificial earthquakes in order to liberate gas and oil E supersaturate the rock with the fracking fluid, which scrubs gas and oil from the rock by abrasion

create cracks within the rock in order to provide pathways for the release of gas and oil

Sustainable use of forests in the United States would likely be encouraged by A cutting small groups of medium- and large- sized trees in uneven-aged forests B clear-cutting old-growth forests to allow for secondary succession C allowing road building in wilderness areas so that older, clear-cut forests have time to regenerate D logging on steep slopes in designated wilderness areas E enforcing all provisions of the Resource Conservation and Recovery Act (RCRA)

cutting small groups of medium- and large- sized trees in uneven-aged forests

One solution for reducing the amount of atmospheric carbon dioxide would be to A increase oceanic temperatures to enhance carbon dioxide uptake B increase the rate of removal of tropical rain forests C decrease the total area of rice paddies D decrease the use of fossil fuels E decrease the production of chlorofluorocarbons

decrease the use of fossil fuels

One solution for reducing the amount of atmospheric carbon dioxide would be to A increase oceanic temperatures to enhance carbon dioxide uptake B increase the rate of removal of tropical rain forests C decrease the total area of rice paddies D decrease the use of fossil fuels E decrease the production of chlorofluorocarbon

decrease the use of fossil fuels

Most of the Earth's deserts are at approximately 30° latitude, north and south, because these latitudes are characterized by A generally warm ocean currents B predominantly low atmospheric pressure C descending dry air currents D slow-moving jet streams E enhanced solar radiation

descending dry air currents

Most of the Earth's deserts are at approximately 30° latitude, north and south, because these latitudes are characterized by A generally warm ocean currents B predominantly low atmospheric pressure C descending dry air currents D slow-moving jet streams E enhanced solar radiation

descending dry air currents

A major limitation of using photovoltaic cells to generate electricity is that they A do not produce as much CO2 as other energy sources do B do not produce as much electricity on cloudy days C have no moving parts D present a danger to birds and bats E cannot be connected to the electrical grid

do not produce as much electricity on cloudy days

One way that consumers can help prevent a decrease in the number of fish species that are experiencing population declines is to A help get laws passed that allow more fishing in areas that are not fished currently, therefore opening access to greater numbers of fish B purchase fish harvested through the use of purse seining and long-lining methods so that only the largest fish are caught C invest in companies that fish near coral reefs, which would take pressure off deep-sea species and eliminate the need for advances in technology D eat fish that are caught using sustainable harvesting methods, therefore ensuring fish populations remain stable and reducing the amount bycatch harvested

eat fish that are caught using sustainable harvesting methods, therefore ensuring fish populations remain stable and reducing the amount bycatch harvested

The consumption of mosquitoes by bats and the control of flooding provided by tropical forests in mountainous areas of Central America are examples of A adaptive radiation B ecosystem services C evolution by natural selection D ecological equilibrium

ecosystem services

The consumption of mosquitoes by bats and the control of flooding provided by tropical forests in mountainous areas of Central America are examples of A adaptive radiation B ecosystem services C evolution by natural selection D ecological equilibrium E positive feedback loops

ecosystem services

Overgrazing of grasslands can lead to reduced range quality. Two of the major effects of overgrazing are A erosion and desertification B higher fire potential and increased productivity C eutrophication and increased methane production D higher primary productivity and ammonification E soil compaction and subsidence

erosion and desertification

Eutrophication

excessive richness of nutrients in a lake or other body of water, frequently due to runoff from the land, which causes a dense growth of plant life and death of animal life from lack of oxygen.

The presence of high levels of fecal coliform bacteria in a water source indicates that the water A is safe to drink B is safe to swim in C contains too little oxygen to support fish life D has been recently chlorinated at a sewage treatment plant E has been contaminated by untreated human or animal waste

has been contaminated by untreated human or animal waste

Although the fertility rate for women in the United States has declined in recent years to a value below replacement level, the United States population is still increasing because of A lower average age at first marriage B lower infant death rates C increased longevity D improved health care E immigration

immigration

A country with an age distribution like that shown in the figure above, is most likely a country that A is experiencing rapid growth B is experiencing slow or no population growth C is experiencing a high death rate D has 40% of the population under age 15 E is a less-developed country

is experiencing slow or no population growth

The CITES treaty has been helpful in protecting endangered animals and plants by A listing all species that can be hunted, traded, and used commercially B listing those species and products whose international trade is controlled C funding projects for breeding endangered plants and animals D preventing the hunting of whales and dolphins E specifying prices for certain plant and animal products

listing those species and products whose international trade is controlled

As urbanization increases and natural soil surfaces are covered, the groundwater supply is reduced due to A increased evaporation and transpiration B decreased surface runoff C loss of recharge area D confinement of aquifers E capping of artesian wells

loss of recharge area

The most abundant gas in Earth's atmosphere is A nitrogen B oxygen C water vapor D carbon dioxide E hydrogen

nitrogen

Nitrogen fixation

nitrogen fixation, any natural or industrial process that causes free nitrogen

Transpiration

of a plant or leaf) the exhalation of water vapor through the stomata.

When a rain forest is slashed and burned, the local concentration of carbon dioxide in the atmosphere increases. This is primarily due to A changes in the local climate B oxidation of carbon compounds C cellular respiration of rain-forest plants D erosion of exposed soil E carbon dioxide being released by anaerobic organisms

oxidation of carbon compounds

There are various stages in the formation of coal as heat and pressure are increased and moisture content is decreased. These stages, in order, are A bituminous, peat, lignite, anthracite B peat, lignite, bituminous, anthracite C peat, lignite, anthracite, bituminous D lignite, bituminous, anthracite, peat E anthracite, bituminous, peat, lignite

peat, lignite, bituminous, anthracite

Directions: Select the choice that best fits each statement. The following question(s) refer to the graph below, which shows different phases related to the birth and death rates for a typical human population over time. Zero population growth is associated with A phase I only B phase II only C phase III only D phase IV only E phases I and IV

phases I and IV

The two major processes involved in the carbon cycle are A weathering and erosion B photosynthesis and respiration C evaporation and transpiration D erosion and deposition E fixation and denitrification

photosynthesis and respiration

Most volcanism in the world is associated with A plate boundaries B midcontinental hot spots C faulting D aquifer depletion E desertification

plate boundaries

Most volcanism in the world is associated with A plate boundaries B midcontinental hot spots C faulting D aquifer depletion E desertification

plate boundaries

The Sun's radiation provides the energy for all of the following EXCEPT A plant growth B ocean currents C hurricanes D plate tectonics E cloud formation

plate tectonics

In contrast to low-level radioactive waste, most high-level radioactive waste is currently A put into steel drums and dumped into the ocean B incinerated C buried in government landfills D recycled E stored at reactor sites

stored at reactor sites

Directions: The following question(s) refer to the graph below showing the percent contribution of major fuel sources that have supplied energy for the United States for the past 150 years. The category labeled "Other" includes nuclear power, hydroelectricity, solar and wind power, and other alternative sources of energy.The primary cause for the decline in the use of wood as a fuel in the eighteen hundreds was the A discovery of major oil fields in the Middle East B lowering of per capita energy consumption C reduced wood availability resulting from widespread deforestation D global rise in temperatures during the 1800s E lower energy density of coal as compared to wood

reduced wood availability resulting from widespread deforestation

Farmers need to make many decisions about how they grow their crops. One of these decisions is whether to use synthetic fertilizer or organic fertilizer, such as manure. An advantage of using synthetic fertilizer instead of organic fertilizer is that A synthetic fertilizer can provide more targeted macronutrients to the crops B synthetic fertilizer remains in the soil longer than organic fertilizer does, reducing the volume needed and, therefore, the cost to the farmer C synthetic fertilizer adds to soil texture and increases the soil's water-holding capacity D organic fertilizer runs off more readily into waterways, leading to nutrient overload

synthetic fertilizer can provide more targeted macronutrients to the crops

The ultimate source of energy for terrestrial ecosystem is A nutrients in soil B nutrients in vegetation C primary consumers D producers E the Sun

the Sun

The most populous countries in the world are China, India, and A Indonesia B Russia C the United States D Japan E Mexico

the United States

Environmentalists oppose the mining of antarctic mineral resources because A territorial claims to Antarctica are unresolved B the existence of valuable mineral deposits in the antarctic environment is unlikely C the antarctic environment is fragile and extremely vulnerable to the disturbances that would accompany development D currently known world reserves of important metals and oils are considered inexhaustible E the demand for minerals is expected to decline as the world's nations become more industrialized

the antarctic environment is fragile and extremely vulnerable to the disturbances that would accompany development

The farming of fish such as salmon at aquaculture facilities poses a threat to marine ecosystems because A the farmed fish can escape and outcompete wild fish for food and territory B farm-raised salmon often pass on toxic chemicals such as mercury to eagles and other fish-eating birds C invasive plant species common in aquaculture facilities can spread to nearby waters D fewer wild fish will be harvested for human consumption

the farmed fish can escape and outcompete wild fish for food and territory

An insect population in an agricultural field is affected by monthly applications of a pesticide, as shown in the graph above. A likely cause of the overall increase in the insect population over time is A an increase in the population of insect predators B an increase in average temperature over the summer C an increase in soil salinity D a decrease in the moisture content of the soil E the survival of increasing numbers of resistant insects

the survival of increasing numbers of resistant insects

Ozone in the stratosphere is important to organisms at Earth's surface because ozone molecules are very strong absorbers of A infrared rays B ultraviolet rays C microwaves D visible light rays E x-rays

ultraviolet rays

A new 450-unit housing development will replace several small farms on the outskirts of Fremont. The development will be composed of 1/4-hectare lots with nearly identical footprints, as show below. One important step that homeowners in the new development can take to protect water quality in Samel Creek would be to A apply lawn fertilizers immediately before storm events B channelize Samel Creek to improve water flow during storm events C use only persistent organic pesticides to control weed species in their lawns D make certain that pet waste is disposed of directly into storm drains E use rain barrels to reduce the flow of storm water into the creek

use rain barrels to reduce the flow of storm water into the creek

Geothermal energy plants are a viable alternative energy source because they A use the heat from Earth's interior to create steam that is used to generate electricity and then can be pumped back into the ground B can be built for use in any location where surface drilling is possible and construction of geothermal plants increases land stability C use the heat from Earth's interior to directly heat buildings and houses and give off no greenhouse gasses D do not give off hazardous air pollutants into the atmosphere and require very little water

use the heat from Earth's interior to create steam that is used to generate electricity and then can be pumped back into the ground B

A sustainable society would emphasize A maintaining the current rates of energy flow and resource use B converting the world's high-quality energy resources to low-quality heat C recycling both matter and high-quality energy D using energy efficiently and reusing and recycling matter E quickly expanding nuclear power, because it is a renewable resource

using energy efficiently and reusing and recycling matter

Most deserts on Earth are located near the 30° north and 30° south latitudes because humidity is lower at 30°. The best explanation for the lower humidity is that A warm air rises at 90° latitude, cools, and sinks at 30° B cool air rises at 30° latitude, warms, and sinks at the equator C warm air rises at the equator, cools, and sinks at 30° latitude D trade winds transfer moisture away from 30° latitude E rain-shadow effects are most pronounced at 30° latitude

warm air rises at the equator, cools, and sinks at 30° latitude

If the air layers in the diagram were relabeled to illustrate normal atmospheric conditions, the correct labeling from the air layer closest to the Earth's surface to the one farthest from the Earth's surface would be A cool air, cooler air, warm air B warm air, cool air, cooler air C cooler air, warm air, cool air D cooler air, cool air, warm air

warm air, cool air, cooler air


Conjuntos de estudio relacionados

Refrigeration Cycle +Refrigeration (4 Major components)

View Set

Factoring Trinomials: a = 1 Assignment

View Set

Lesson 12: Female Reproductive System

View Set

FIN3403 - Exam One Review Questions (Chp 3)

View Set

Human Resource Management: Chapter 4

View Set